[obm-l] Fatoração

2022-09-14 Por tôpico Daniel Quevedo
A soma dos valores inteiros de a para os quais (x -10)(x+a) +1 seja
faturável num produto (x+b)(x+c) com b e c inteiros é:

A) 8
B) 10
C) 12
D) 20
E) 24

Resp: D
-- 
Fiscal: Daniel Quevedo

-- 
Esta mensagem foi verificada pelo sistema de antiv�rus e
 acredita-se estar livre de perigo.



[obm-l] Re: [obm-l] Re: [obm-l] Re: [obm-l] Fatoração prima de n!

2018-12-29 Por tôpico Artur Costa Steiner
Sim, porque, se o primo p satisfizer a tais condições, então, para k >= 2,
p^k >= n. Logo, se p estiver na fatoração de n!, p tem expoente 1.

Artur

Em sáb, 29 de dez de 2018 16:58, Pedro José  Boa tarde!
> Na verdade: n/2 >= [raiz(n)].
> Mas vale da mesma forma.
>
> Saudações,
> PJMS
>
> Em sáb, 29 de dez de 2018 13:36, Pedro José 
>> Bom dia!
>> Com o teorema mencionado dá para mostrar que existe pelo menos um primo
>> >=[raiz(n) +1] e <= n.
>> Para n = 2 ou n =3 é imediato.
>> para n>=4: n/2>= raiz(n) >=[raiz(n)] + 1.
>> Vou dar uma olhada no Wikipedia. Não conhecia esse teorema.
>> Mas só para tirar uma dúvida, está correto afirmar que ocorrerá para
>> qualquer fator p que seja maior ou igual que [raiz(n)+1] e menor ou ogual
>> que n?
>> Saudações,
>> PJMS
>>
>> Em qui, 27 de dez de 2018 21:03, Claudio Buffara <
>> claudio.buff...@gmail.com escreveu:
>>
>>> Médio... vê na Wikipedia
>>>
>>> Enviado do meu iPhone
>>>
>>> Em 27 de dez de 2018, à(s) 14:24, Artur Steiner <
>>> artur.costa.stei...@gmail.com> escreveu:
>>>
>>> Obrigado a todos.
>>>
>>> Tinha esquecido do que é atualmente o teorema de Bertrand. A
>>> demonstração é muito complicada?
>>>
>>> Artur Costa Steiner
>>>
>>> Em qui, 27 de dez de 2018 00:38, Claudio Buffara <
>>> claudio.buff...@gmail.com escreveu:
>>>
 É o maior primo <= n.
 Pelo teorema (“postulado†) de Bertrand (se p é primo, então
 existe um primo q tal que p < q < 2p).

 Enviado do meu iPhone

 Em 26 de dez de 2018, Ã (s) 19:44, Artur Steiner <
 artur.costa.stei...@gmail.com> escreveu:

 > Mostre que, para n >= 2, a fatoração prima de n! contém um
 fator com expoente 1.
 >
 > Abraços.
 >
 > Artur Costa Steiner
 >
 > --
 > Esta mensagem foi verificada pelo sistema de antivírus e
 > acredita-se estar livre de perigo.

 --
 Esta mensagem foi verificada pelo sistema de antivírus e
 Â acredita-se estar livre de perigo.



 =
 Instru�ões para entrar na lista, sair da lista e usar a lista em
 http://www.mat.puc-rio.br/~obmlistas/obm-l.html

 =

>>>
>>> --
>>> Esta mensagem foi verificada pelo sistema de antivírus e
>>> acredita-se estar livre de perigo.
>>>
>>>
>>> --
>>> Esta mensagem foi verificada pelo sistema de antivírus e
>>> acredita-se estar livre de perigo.
>>>
>>
> --
> Esta mensagem foi verificada pelo sistema de antivírus e
> acredita-se estar livre de perigo.

-- 
Esta mensagem foi verificada pelo sistema de antiv�rus e
 acredita-se estar livre de perigo.



[obm-l] Re: [obm-l] Re: [obm-l] Fatoração prima de n!

2018-12-29 Por tôpico Pedro José
Boa tarde!
Na verdade: n/2 >= [raiz(n)].
Mas vale da mesma forma.

Saudações,
PJMS

Em sáb, 29 de dez de 2018 13:36, Pedro José  Bom dia!
> Com o teorema mencionado dá para mostrar que existe pelo menos um primo
> >=[raiz(n) +1] e <= n.
> Para n = 2 ou n =3 é imediato.
> para n>=4: n/2>= raiz(n) >=[raiz(n)] + 1.
> Vou dar uma olhada no Wikipedia. Não conhecia esse teorema.
> Mas só para tirar uma dúvida, está correto afirmar que ocorrerá para
> qualquer fator p que seja maior ou igual que [raiz(n)+1] e menor ou ogual
> que n?
> Saudações,
> PJMS
>
> Em qui, 27 de dez de 2018 21:03, Claudio Buffara <
> claudio.buff...@gmail.com escreveu:
>
>> Médio... vê na Wikipedia
>>
>> Enviado do meu iPhone
>>
>> Em 27 de dez de 2018, à(s) 14:24, Artur Steiner <
>> artur.costa.stei...@gmail.com> escreveu:
>>
>> Obrigado a todos.
>>
>> Tinha esquecido do que é atualmente o teorema de Bertrand. A
>> demonstração é muito complicada?
>>
>> Artur Costa Steiner
>>
>> Em qui, 27 de dez de 2018 00:38, Claudio Buffara <
>> claudio.buff...@gmail.com escreveu:
>>
>>> É o maior primo <= n.
>>> Pelo teorema (“postulado†) de Bertrand (se p é primo, então existe
>>> um primo q tal que p < q < 2p).
>>>
>>> Enviado do meu iPhone
>>>
>>> Em 26 de dez de 2018, Ã (s) 19:44, Artur Steiner <
>>> artur.costa.stei...@gmail.com> escreveu:
>>>
>>> > Mostre que, para n >= 2, a fatoração prima de n! contém um
>>> fator com expoente 1.
>>> >
>>> > Abraços.
>>> >
>>> > Artur Costa Steiner
>>> >
>>> > --
>>> > Esta mensagem foi verificada pelo sistema de antivírus e
>>> > acredita-se estar livre de perigo.
>>>
>>> --
>>> Esta mensagem foi verificada pelo sistema de antivírus e
>>> Â acredita-se estar livre de perigo.
>>>
>>>
>>> =
>>> Instru�ões para entrar na lista, sair da lista e usar a lista em
>>> http://www.mat.puc-rio.br/~obmlistas/obm-l.html
>>> =
>>>
>>
>> --
>> Esta mensagem foi verificada pelo sistema de antivírus e
>> acredita-se estar livre de perigo.
>>
>>
>> --
>> Esta mensagem foi verificada pelo sistema de antivírus e
>> acredita-se estar livre de perigo.
>>
>

-- 
Esta mensagem foi verificada pelo sistema de antiv�rus e
 acredita-se estar livre de perigo.



Re: [obm-l] Re: [obm-l] Re: [obm-l] Fatoração prima de n!

2018-12-29 Por tôpico Artur_steiner
Sim, porque se o primo p satisfizer a esta condição, então, para todo k >= 2, 
temos p^k > n. Logo, se p aparecer na fatoração de n!, será com expoente 1.
Artur


Enviado do meu Samsung Mobile da Claro
 Mensagem original De: Pedro José  Data: 
29/12/2018  13:36  (GMT-03:00) Para: obm-l@mat.puc-rio.br Assunto: [obm-l] Re: 
[obm-l] Re: [obm-l] Fatoração prima de n! 
Bom dia!Com o teorema mencionado dá para mostrar que existe pelo menos um primo 
>=[raiz(n) +1] e <= n.Para n = 2 ou n =3 é imediato.para n>=4: n/2>= raiz(n) 
>=[raiz(n)] + 1. Vou dar uma olhada no Wikipedia. Não conhecia esse teorema.Mas 
só para tirar uma dúvida, está correto afirmar que ocorrerá para qualquer fator 
p que seja maior ou igual que [raiz(n)+1] e menor ou ogual que n?Saudações, PJMS

Em qui, 27 de dez de 2018 21:03, Claudio Buffara  
escreveu:

Obrigado a todos.
Tinha esquecido do que é atualmente o teorema de Bertrand. A demonstração é 
muito complicada?

Artur Costa Steiner
Em qui, 27 de dez de 2018 00:38, Claudio Buffara  escreveu:



> Mostre que, para n >= 2, a fatoração prima de n! contém um fator com 
> expoente 1.

> 

> Abraços.

> 

> Artur Costa Steiner

> 

> -- 

> Esta mensagem foi verificada pelo sistema de antivírus e 

> acredita-se estar livre de perigo.



-- 

Esta mensagem foi verificada pelo sistema de antivírus e

 acredita-se estar livre de perigo.





=

Instru�ões para entrar na lista, sair da lista e usar a lista em

http://www.mat.puc-rio.br/~obmlistas/obm-l.html

=



--

Esta mensagem foi verificada pelo sistema de antivírus e 

 acredita-se estar livre de perigo.


--

Esta mensagem foi verificada pelo sistema de antivírus e 

 acredita-se estar livre de perigo.



--

Esta mensagem foi verificada pelo sistema de antiv�rus e 

 acredita-se estar livre de perigo.


-- 
Esta mensagem foi verificada pelo sistema de antiv�rus e
 acredita-se estar livre de perigo.



[obm-l] Re: [obm-l] Re: [obm-l] Fatoração prima de n!

2018-12-29 Por tôpico Pedro José
Bom dia!
Com o teorema mencionado dá para mostrar que existe pelo menos um primo
>=[raiz(n) +1] e <= n.
Para n = 2 ou n =3 é imediato.
para n>=4: n/2>= raiz(n) >=[raiz(n)] + 1.
Vou dar uma olhada no Wikipedia. Não conhecia esse teorema.
Mas só para tirar uma dúvida, está correto afirmar que ocorrerá para
qualquer fator p que seja maior ou igual que [raiz(n)+1] e menor ou ogual
que n?
Saudações,
PJMS

Em qui, 27 de dez de 2018 21:03, Claudio Buffara  Médio... vê na Wikipedia
>
> Enviado do meu iPhone
>
> Em 27 de dez de 2018, à(s) 14:24, Artur Steiner <
> artur.costa.stei...@gmail.com> escreveu:
>
> Obrigado a todos.
>
> Tinha esquecido do que é atualmente o teorema de Bertrand. A
> demonstração é muito complicada?
>
> Artur Costa Steiner
>
> Em qui, 27 de dez de 2018 00:38, Claudio Buffara <
> claudio.buff...@gmail.com escreveu:
>
>> É o maior primo <= n.
>> Pelo teorema (“postulado†) de Bertrand (se p é primo, então existe
>> um primo q tal que p < q < 2p).
>>
>> Enviado do meu iPhone
>>
>> Em 26 de dez de 2018, Ã (s) 19:44, Artur Steiner <
>> artur.costa.stei...@gmail.com> escreveu:
>>
>> > Mostre que, para n >= 2, a fatoração prima de n! contém um
>> fator com expoente 1.
>> >
>> > Abraços.
>> >
>> > Artur Costa Steiner
>> >
>> > --
>> > Esta mensagem foi verificada pelo sistema de antivírus e
>> > acredita-se estar livre de perigo.
>>
>> --
>> Esta mensagem foi verificada pelo sistema de antivírus e
>> Â acredita-se estar livre de perigo.
>>
>>
>> =
>> Instru�ões para entrar na lista, sair da lista e usar a lista em
>> http://www.mat.puc-rio.br/~obmlistas/obm-l.html
>> =
>>
>
> --
> Esta mensagem foi verificada pelo sistema de antivírus e
> acredita-se estar livre de perigo.
>
>
> --
> Esta mensagem foi verificada pelo sistema de antivírus e
> acredita-se estar livre de perigo.
>

-- 
Esta mensagem foi verificada pelo sistema de antiv�rus e
 acredita-se estar livre de perigo.



Re: [obm-l] Re: [obm-l] Fatoração prima de n!

2018-12-27 Por tôpico Claudio Buffara
Médio... vê na Wikipedia 

Enviado do meu iPhone

Em 27 de dez de 2018, à(s) 14:24, Artur Steiner  
escreveu:

> Obrigado a todos.
> 
> Tinha esquecido do que é atualmente o teorema de Bertrand. A demonstração 
> é muito complicada?
> 
> Artur Costa Steiner
> 
> Em qui, 27 de dez de 2018 00:38, Claudio Buffara  escreveu:
>> É o maior primo <= n.
>> Pelo teorema (“postulado”) de Bertrand (se p é primo, então existe um 
>> primo q tal que p < q < 2p).
>> 
>> Enviado do meu iPhone
>> 
>> Em 26 de dez de 2018, Ã (s) 19:44, Artur Steiner 
>>  escreveu:
>> 
>> > Mostre que, para n >= 2, a fatoração prima de n! contém um fator 
>> > com expoente 1.
>> > 
>> > Abraços.
>> > 
>> > Artur Costa Steiner
>> > 
>> > -- 
>> > Esta mensagem foi verificada pelo sistema de antivírus e 
>> > acredita-se estar livre de perigo.
>> 
>> -- 
>> Esta mensagem foi verificada pelo sistema de antivírus e
>> Â acredita-se estar livre de perigo.
>> 
>> 
>> =
>> Instru�ões para entrar na lista, sair da lista e usar a lista em
>> http://www.mat.puc-rio.br/~obmlistas/obm-l.html
>> =
> 
> -- 
> Esta mensagem foi verificada pelo sistema de antivírus e 
> acredita-se estar livre de perigo.

-- 
Esta mensagem foi verificada pelo sistema de antiv�rus e
 acredita-se estar livre de perigo.



[obm-l] Re: [obm-l] Fatoração prima de n!

2018-12-27 Por tôpico Artur Steiner
Obrigado a todos.

Tinha esquecido do que é atualmente o teorema de Bertrand. A demonstração é
muito complicada?

Artur Costa Steiner

Em qui, 27 de dez de 2018 00:38, Claudio Buffara  É o maior primo <= n.
> Pelo teorema (“postulado”) de Bertrand (se p é primo, então existe um
> primo q tal que p < q < 2p).
>
> Enviado do meu iPhone
>
> Em 26 de dez de 2018, à(s) 19:44, Artur Steiner <
> artur.costa.stei...@gmail.com> escreveu:
>
> > Mostre que, para n >= 2, a fatoração prima de n! contém um fator com
> expoente 1.
> >
> > Abraços.
> >
> > Artur Costa Steiner
> >
> > --
> > Esta mensagem foi verificada pelo sistema de antivírus e
> > acredita-se estar livre de perigo.
>
> --
> Esta mensagem foi verificada pelo sistema de antivírus e
>  acredita-se estar livre de perigo.
>
>
> =
> Instru�ões para entrar na lista, sair da lista e usar a lista em
> http://www.mat.puc-rio.br/~obmlistas/obm-l.html
> =
>

-- 
Esta mensagem foi verificada pelo sistema de antiv�rus e
 acredita-se estar livre de perigo.



[obm-l] Re: [obm-l] Fatoração prima de n!

2018-12-27 Por tôpico Pedro José
Boa tarde!

Não sei como provar que existe pelo menos um primop tq n >= p >= [raiz(n)]
+1.
Mas na verdade todos os primos p, tq tq n >= p >= [raiz(n)] +1, terão
expoente =1.
Onde [x] = parte inteira de x.

Sds,
PJMS

Em qui, 27 de dez de 2018 às 00:38, Claudio Buffara <
claudio.buff...@gmail.com> escreveu:

> É o maior primo <= n.
> Pelo teorema (“postulado”) de Bertrand (se p é primo, então existe um
> primo q tal que p < q < 2p).
>
> Enviado do meu iPhone
>
> Em 26 de dez de 2018, à(s) 19:44, Artur Steiner <
> artur.costa.stei...@gmail.com> escreveu:
>
> > Mostre que, para n >= 2, a fatoração prima de n! contém um fator com
> expoente 1.
> >
> > Abraços.
> >
> > Artur Costa Steiner
> >
> > --
> > Esta mensagem foi verificada pelo sistema de antivírus e
> > acredita-se estar livre de perigo.
>
> --
> Esta mensagem foi verificada pelo sistema de antivírus e
>  acredita-se estar livre de perigo.
>
>
> =
> Instru�ões para entrar na lista, sair da lista e usar a lista em
> http://www.mat.puc-rio.br/~obmlistas/obm-l.html
> =
>

-- 
Esta mensagem foi verificada pelo sistema de antiv�rus e
 acredita-se estar livre de perigo.



Re: [obm-l] Fatoração prima de n!

2018-12-26 Por tôpico Claudio Buffara
É o maior primo <= n.
Pelo teorema (“postulado”) de Bertrand (se p é primo, então existe um primo q 
tal que p < q < 2p).

Enviado do meu iPhone

Em 26 de dez de 2018, à(s) 19:44, Artur Steiner  
escreveu:

> Mostre que, para n >= 2, a fatoração prima de n! contém um fator com 
> expoente 1.
> 
> Abraços.
> 
> Artur Costa Steiner
> 
> -- 
> Esta mensagem foi verificada pelo sistema de antivírus e 
> acredita-se estar livre de perigo.

-- 
Esta mensagem foi verificada pelo sistema de antiv�rus e
 acredita-se estar livre de perigo.


=
Instru��es para entrar na lista, sair da lista e usar a lista em
http://www.mat.puc-rio.br/~obmlistas/obm-l.html
=


[obm-l] Fatoração prima de n!

2018-12-26 Por tôpico Artur Steiner
Mostre que, para n >= 2, a fatoração prima de n! contém um fator com
expoente 1.

Abraços.

Artur Costa Steiner

-- 
Esta mensagem foi verificada pelo sistema de antiv�rus e
 acredita-se estar livre de perigo.



[obm-l] Re: [obm-l] fatoração

2017-09-14 Por tôpico ilhadepaqueta

Caro Amigo Carlos Nehab, obrigado pela explicação.

Você poderia me indicar um livro que tivesse bastante fatoração e produtos 
notáveis?


Abraços
Hermann


Oi,
x3 + x2y + x2y + x2y + xy2 + xy2 + xy2 + y3
= (x3 + x2y) + 2(x2y+xy2) + (xy2 + y3)
=  x2*(x+y)* + 2xy*(x+y)* + y2*(x+y) *
= (x2+2xy+y2)(x+y) = (x+y)3...
The end...


From: ilhadepaqu...@bol.com.br
Sent: Tuesday, September 12, 2017 2:23 PM
To: obm-l@mat.puc-rio.br
Subject: [obm-l] fatoração

Meus amigos, por favor,  como fatorar  (agrupando!?) x^3 + 3x^2y + 3xy^2 + 
y^3 e chegar em (x+y)^3 ?


(x+y)^3=x^3 + 3x^2y+3xy^2+y^3

Perdoem –me !

Abraços

Hermann

--
Esta mensagem foi verificada pelo sistema de antiv�rus e
acredita-se estar livre de perigo. 



--
Esta mensagem foi verificada pelo sistema de antiv�rus e
acredita-se estar livre de perigo.

=
Instru��es para entrar na lista, sair da lista e usar a lista em
http://www.mat.puc-rio.br/~obmlistas/obm-l.html
=


[obm-l] Re: [obm-l] fatoração

2017-09-12 Por tôpico Carlos Nehab
Oi,
x3 + x2y + x2y + x2y + xy2 + xy2 + xy2 + y3
= (x3 + x2y) + 2(x2y+xy2) + (xy2 + y3)
=  x2*(x+y)* + 2xy*(x+y)* + y2*(x+y) *
= (x2+2xy+y2)(x+y) = (x+y)3...
The end...

Em 12 de setembro de 2017 14:23,  escreveu:

> Meus amigos, por favor,  como fatorar  (agrupando!?) x^3 + 3x^2y + 3xy^2 +
> y^3 e chegar em (x+y)^3 ?
>
> (x+y)^3=x^3 + 3x^2y+3xy^2+y^3
>
> Perdoem –me !
>
> Abraços
>
> Hermann
>
> --
> Esta mensagem foi verificada pelo sistema de antivírus e
> acredita-se estar livre de perigo.
>

-- 
Esta mensagem foi verificada pelo sistema de antiv�rus e
 acredita-se estar livre de perigo.



[obm-l] fatoração

2017-09-12 Por tôpico ilhadepaqueta
Meus amigos, por favor,  como fatorar  (agrupando!?) x^3 + 3x^2y + 3xy^2 + y^3 
e chegar em (x+y)^3 ?

(x+y)^3=x^3 + 3x^2y+3xy^2+y^3

Perdoem –me !

Abraços

Hermann
-- 
Esta mensagem foi verificada pelo sistema de antiv�rus e
 acredita-se estar livre de perigo.



[obm-l] Re: [obm-l] RE: [obm-l] fatoração

2013-05-17 Por tôpico terence thirteen
Mas isto não é matar mosquito com bazuca?


Em 15 de maio de 2013 23:29, João Maldonado
joao_maldona...@hotmail.comescreveu:

 Um jeito que sempre funciona é usar a fatoração de ferrari. Ela resolve
 qualquer equação de 4 grau fatorando-a em duas equações de segundo grau.
 Não é sempre que os coeficientes são inteiros ou racionais, mas nesse caso
 (como você já viu na resposta) eles são.

 Primeiramente deve-se deixar o coefiente de quarto grau como 1, já fizemos
 isso
 Depois devemos mudar a variável para cancelar o termo de terceiro grau.
 Dada a equação x^4 + ax³ + bx² + cx + d=0
 Escolha x=(y-a/4)
 No nosso caso x=(y-1/2)
 Resulta em: y^4 -5/2 y² -4y -7/16=0

 Agora rearrange os termos: Termo do quarto grau do lado esquerdo e os
 demais do lado direito.
 Some Ay² +B de ambos os lados para construir dois quadrados perfeitos (um
 quadrado perfeito tem delta=0)

 y^4 + Ay² + B = (A+5/2)y² + 4y + (B+7/16)
 Temos
 A² = 4B
 16 = 4(A+5/2)(B+7/16) (multiplicando por 32)

 128 = (2A+5)(16B+7)
 128 = (2A+5)(4A²+7)

 Agora resolvemos a equação do terceiro grau (no caso geral se resolve por
 cardano, mas como sabemos que as raízes são racionais, resolvemos pelo
 teorema das raízes racionais)
 A=3/2
 Desse modo B = 9/16

 E achamos: (y²+3/4)² = (2y + 1)²
 (y² - 2y - 1/4)(y² + 2y +7/4) = 0
 Substituindo (x² - x - 1)(x² + 3x + 3) = 0

 Abraço

 João

 --
 Date: Wed, 15 May 2013 16:47:23 -0300
 Subject: [obm-l] fatoração
 From: oliho...@gmail.com
 To: obm-l@mat.puc-rio.br

 O polinômio p(x) = x^4 + 2x^3 - x^2 - 6x - 3
 se fatora como p(x) = (x^2  - x - 1).(x^2 + 3x + 3)
 Alguém poderia me ajudar em como chegar a essa fatoração?
 Agradeço a ajuda.





-- 
/**/
神が祝福

Torres


[obm-l] fatoração

2013-05-15 Por tôpico Mauricio Barbosa
O polinômio p(x) = x^4 + 2x^3 - x^2 - 6x - 3
se fatora como p(x) = (x^2  - x - 1).(x^2 + 3x + 3)
Alguém poderia me ajudar em como chegar a essa fatoração?
Agradeço a ajuda.


[obm-l] Re: [obm-l] fatoração

2013-05-15 Por tôpico terence thirteen
Uma ideia inicial seria tentar raízes racionais - acho que não vai
funcionar. Depois disso, resta tentar a sorte com P(x)=(x^2-px+q)(x^2-rx+s)
e ter um pouquinho de fé...

Talvez outra ideia seria tentar algo relacionado a raízes da unidade, mas
não vou arriscar...


Em 15 de maio de 2013 16:47, Mauricio Barbosa oliho...@gmail.com escreveu:

 O polinômio p(x) = x^4 + 2x^3 - x^2 - 6x - 3
 se fatora como p(x) = (x^2  - x - 1).(x^2 + 3x + 3)
 Alguém poderia me ajudar em como chegar a essa fatoração?
 Agradeço a ajuda.





-- 
/**/
神が祝福

Torres


[obm-l] RE: [obm-l] fatoração

2013-05-15 Por tôpico João Maldonado
Um jeito que sempre funciona é usar a fatoração de ferrari. Ela resolve 
qualquer equação de 4 grau fatorando-a em duas equações de segundo grau. Não é 
sempre que os coeficientes são inteiros ou racionais, mas nesse caso (como você 
já viu na resposta) eles são.

Primeiramente deve-se deixar o coefiente de quarto grau como 1, já fizemos isso
Depois devemos mudar a variável para cancelar o termo de terceiro grau.
Dada a equação x^4 + ax³ + bx² + cx + d=0
Escolha x=(y-a/4)
No nosso caso x=(y-1/2)
Resulta em: y^4 -5/2 y² -4y -7/16=0

Agora rearrange os termos: Termo do quarto grau do lado esquerdo e os demais do 
lado direito.
Some Ay² +B de ambos os lados para construir dois quadrados perfeitos (um 
quadrado perfeito tem delta=0)

y^4 + Ay² + B = (A+5/2)y² + 4y + (B+7/16)
Temos
A² = 4B
16 = 4(A+5/2)(B+7/16) (multiplicando por 32)

128 = (2A+5)(16B+7)
128 = (2A+5)(4A²+7)

Agora resolvemos a equação do terceiro grau (no caso geral se resolve por 
cardano, mas como sabemos que as raízes são racionais, resolvemos pelo teorema 
das raízes racionais)
A=3/2
Desse modo B = 9/16

E achamos: (y²+3/4)² = (2y + 1)²
(y² - 2y - 1/4)(y² + 2y +7/4) = 0
Substituindo (x² - x - 1)(x² + 3x + 3) = 0

Abraço

João

Date: Wed, 15 May 2013 16:47:23 -0300
Subject: [obm-l] fatoração
From: oliho...@gmail.com
To: obm-l@mat.puc-rio.br

O polinômio p(x) = x^4 + 2x^3 - x^2 - 6x - 3se fatora como p(x) = (x^2  - x - 
1).(x^2 + 3x + 3)Alguém poderia me ajudar em como chegar a essa 
fatoração?Agradeço a ajuda.


  

[obm-l] Fatoração(?)

2013-02-11 Por tôpico marcone augusto araújo borges

Se a^3 + b^3 +  c^3 + d^3 = a + b + c + d = o,mostre que a soma de dois desses 
números é zero.
  

[obm-l] RE: [obm-l] Fatoração(?)

2013-02-11 Por tôpico João Maldonado

(a+b)( (a+b)²-3ab ) + (c+d)( (c+d)² -3cd) = 0
(a+b) = -(c+d)
(a+b)( (a+b)²-3ab ) = (a+b)( (c+d)²-3cd )

1) Ou (a+b) = 0

2) Ou ab=cd
Desse modo
c+d = -(a+b)
cd = ab

Gera uma equação do segundo grau - (c,d) = (-a, -b)
Desse modo
c+a = 0 ou c+b = 0

CQD


From: marconeborge...@hotmail.com
To: obm-l@mat.puc-rio.br
Subject: [obm-l] Fatoração(?)
Date: Mon, 11 Feb 2013 23:04:17 +





Se a^3 + b^3 +  c^3 + d^3 = a + b + c + d = o,mostre que a soma de dois desses 
números é zero.

  

[obm-l] RE: [obm-l] Re: [obm-l] RE: [obm-l] RE: [obm-l] fatoração de polinômio

2011-10-12 Por tôpico Luan Gabriel


É, o jeito braçal,depois de muito treino, acaba funcionando na maioria das 
questões... a dúvida quanto a isso era apenas formalismo mesmo, já que de 
antemão dá p desconfiar que o polinômio vai ser fatorado apenas com 
coeficientes inteiros (a questão simplesmente já pedia para fatorar). Tenta 
fatorar no braço x^3+7x^2+7x+14 pra ver. 

Re: [obm-l] RE: [obm-l] fatoração de polinômio

2011-10-11 Por tôpico Paulo César
Some e subtraia x^2. Fica assim:
 x^5-x^2+x^2+x+1=x^2(x^3-1)+x^2+x+1=x^2(x-1)(x^2+x+1)+x^2+x+1= 
(x^2+x+1)(x^3-x^2+1)
Essa é das antigas, do livro Álgebra 1, do Wagner e do Morgado. Esse tipo de 
fatoração é muito difícil. Somar e subtrair coisas costuma dar muita dor de 
cabeça até que se descubra o que fazer.

Um abraço.

Paulo Cesar Sampaio Jr.
Enviado via iPad

Em 11/10/2011, às 00:34, João Maldonado joao_maldona...@hotmail.com escreveu:

 
 Eu faria assim,   
 
 x^5+ x  + 1 = 
 T=x^5 + x^3 + x  + (1-x^3)=
 x(x^4 + x² + 1) + (1-x)(x²+x+1)
 
 Fazendo k =(x^4 + x² + 1)
 y=x²
 temos  k=y²+y+1 =   
 (y³-1)/(y-1)=(x^6-1)/(x²-1)=(x³-1)(x³+1)/(x+1)(x-1)=(x²-x+1)(x²+x+1)
 Logo   
 
 T=(x²+x+1)(x³-x²+x)  + (1-x)(x²+x+1)
 T=(x²+x+1)(x³-x²+1)
 
 []'s
 João
 
 From: luan_gabrie...@hotmail.com
 To: obm-l@mat.puc-rio.br
 Subject: [obm-l] fatoração de polinômio
 Date: Tue, 11 Oct 2011 03:57:55 +0300
 
 Boa noite, entrei hoje na lista,espero ter mandado pro e-mail certo. A 
 questão é encontrar uma fatoração para o polinômio:
  X^5+X+1 
 
 Agradeço a ajuda.


[obm-l] RE: [obm-l] RE: [obm-l] fatoração de polinômio

2011-10-11 Por tôpico Luan Gabriel

Vlw galera!

CC: obm-l@mat.puc-rio.br
From: pcesa...@gmail.com
Subject: Re: [obm-l] RE: [obm-l] fatoração de polinômio
Date: Tue, 11 Oct 2011 06:19:34 -0300
To: obm-l@mat.puc-rio.br

Some e subtraia x^2. Fica assim: 
x^5-x^2+x^2+x+1=x^2(x^3-1)+x^2+x+1=x^2(x-1)(x^2+x+1)+x^2+x+1= 
(x^2+x+1)(x^3-x^2+1)Essa é das antigas, do livro Álgebra 1, do Wagner e do 
Morgado. Esse tipo de fatoração é muito difícil. Somar e subtrair coisas 
costuma dar muita dor de cabeça até que se descubra o que fazer.

Um abraço.
Paulo Cesar Sampaio Jr.Enviado via iPad
Em 11/10/2011, às 00:34, João Maldonado joao_maldona...@hotmail.com escreveu:



Eu faria assim,   
x^5+ x  + 1 = T=x^5 + x^3 + x  + (1-x^3)=x(x^4 + x² + 1) + (1-x)(x²+x+1)
Fazendo k =(x^4 + x² + 1)y=x²temos  k=y²+y+1 =   
(y³-1)/(y-1)=(x^6-1)/(x²-1)=(x³-1)(x³+1)/(x+1)(x-1)=(x²-x+1)(x²+x+1)Logo   
T=(x²+x+1)(x³-x²+x)  + (1-x)(x²+x+1)T=(x²+x+1)(x³-x²+1)
[]'sJoão
From: luan_gabrie...@hotmail.com
To: obm-l@mat.puc-rio.br
Subject: [obm-l] fatoração de polinômio
Date: Tue, 11 Oct 2011 03:57:55 +0300








Boa noite, entrei hoje na lista,espero ter mandado pro e-mail certo. A questão 
é encontrar uma fatoração para o polinômio: X^5+X+1 
Agradeço a ajuda.   
  
  

[obm-l] Re: [obm-l] RE: [obm-l] RE: [obm-l] fatoração de polinômio

2011-10-11 Por tôpico terence thirteen
Outra maneira é tentar uma raiz cúbica da unidade.

Me respondam uma coisa: por que raios vocês tentam demonstrar que o
polinômiuo é redutível, e depois é que vão fatorá-lo?  Não é melhor
fatorar de uma vez?

E ainda prefiro a solução braçal. Ficar epnsando em sacadinhas mágicas
não é meu esporte favorito...

Em 11/10/11, Luan Gabrielluan_gabrie...@hotmail.com escreveu:

 Vlw galera!

 CC: obm-l@mat.puc-rio.br
 From: pcesa...@gmail.com
 Subject: Re: [obm-l] RE: [obm-l] fatoração de polinômio
 Date: Tue, 11 Oct 2011 06:19:34 -0300
 To: obm-l@mat.puc-rio.br

 Some e subtraia x^2. Fica assim:
 x^5-x^2+x^2+x+1=x^2(x^3-1)+x^2+x+1=x^2(x-1)(x^2+x+1)+x^2+x+1=
 (x^2+x+1)(x^3-x^2+1)Essa é das antigas, do livro Álgebra 1, do Wagner e do
 Morgado. Esse tipo de fatoração é muito difícil. Somar e subtrair coisas
 costuma dar muita dor de cabeça até que se descubra o que fazer.

 Um abraço.
 Paulo Cesar Sampaio Jr.Enviado via iPad
 Em 11/10/2011, às 00:34, João Maldonado joao_maldona...@hotmail.com
 escreveu:



 Eu faria assim,
 x^5+ x  + 1 = T=x^5 + x^3 + x  + (1-x^3)=x(x^4 + x² + 1) + (1-x)(x²+x+1)
 Fazendo k =(x^4 + x² + 1)y=x²temos  k=y²+y+1 =
 (y³-1)/(y-1)=(x^6-1)/(x²-1)=(x³-1)(x³+1)/(x+1)(x-1)=(x²-x+1)(x²+x+1)Logo

 T=(x²+x+1)(x³-x²+x)  + (1-x)(x²+x+1)T=(x²+x+1)(x³-x²+1)
 []'sJoão
 From: luan_gabrie...@hotmail.com
 To: obm-l@mat.puc-rio.br
 Subject: [obm-l] fatoração de polinômio
 Date: Tue, 11 Oct 2011 03:57:55 +0300








 Boa noite, entrei hoje na lista,espero ter mandado pro e-mail certo. A
 questão é encontrar uma fatoração para o polinômio: X^5+X+1
 Agradeço a ajuda. 
 
   


-- 
/**/
神が祝福

Torres

=
Instru��es para entrar na lista, sair da lista e usar a lista em
http://www.mat.puc-rio.br/~obmlistas/obm-l.html
=


[obm-l] fatoração de polinômio

2011-10-10 Por tôpico Luan Gabriel

Boa noite, entrei hoje na lista,espero ter mandado pro e-mail certo. A questão 
é encontrar uma fatoração para o polinômio: X^5+X+1 
Agradeço a ajuda. 

[obm-l] Re: [obm-l] fatoração de polinômio

2011-10-10 Por tôpico Pedro Angelo
Opa,

para cálculos mecânicos porém chatos, um site excelente é o Wolfram Alpha.
você coloca o polinômio (ou qualquer coisa computável), e ele te dá
informações sobre a coisa.

por exemplo, se você  coloca um polinômio, ele te diz as raízes, as
fatorações possíveis, o gráfico, etc.
Se você coloca uma matriz, ele te diz informações sobre os
autovalores, o determinante, o traço, etc.
Se você coloca o nome de um país, ele te dá a data de independência, a
quantidade de habitantes, a renda per capita, etc.

http://www.wolframalpha.com/input/?i=X%5E5%2BX%2B1+

Me desculpem se a minha mensagem parece muito propaganda,
principalmente por que eu quase nunca escrevo para a lista, mas eu
acho que é uma ferramenta realmente útil. E ele tem uma opção de show
steps, então você pode ver como ele chegou a cada resultado, o que
torna ele um pouco mais didático.

abraços

2011/10/10 Luan Gabriel luan_gabrie...@hotmail.com:
 Boa noite, entrei hoje na lista,espero ter mandado pro e-mail certo. A
 questão é encontrar uma fatoração para o polinômio:
  X^5+X+1
 Agradeço a ajuda.

=
Instruções para entrar na lista, sair da lista e usar a lista em
http://www.mat.puc-rio.br/~obmlistas/obm-l.html
=


[obm-l] Re: [obm-l] fatoração de polinômio

2011-10-10 Por tôpico Pedro Nascimento
sempre tem o wolfram alpha,
http://www.wolframalpha.com/input/?i=+X^5%2BX%2B1+ , mas nao sei se eh esse
o objetivo

Em 10 de outubro de 2011 21:57, Luan Gabriel
luan_gabrie...@hotmail.comescreveu:

  Boa noite, entrei hoje na lista,espero ter mandado pro e-mail certo. A
 questão é encontrar uma fatoração para o polinômio:
  X^5+X+1

 Agradeço a ajuda.



[obm-l] RE: [obm-l] Re: [obm-l] fatoração de polinômio

2011-10-10 Por tôpico Luan Gabriel

Olhei o site, e realmente é muito bom. Quanto ao problema, ele não apresenta 
uma maneira prática de fatoração;pelo contrário, usa algo muito bizarro. De 
qualquer forma, vi a forma fatorada e,como era de se esperar, ele é redutível 
nos Z e a fatoração resulta em dois polinômios primitivos. Tentei provar que o 
polinômio inicial era redutível nos Z,mas não consegui. Então,não sei se a 
suposição de que o polinômio pode ser fatorado em (X^3+aX^2+bX+1).(X^2+cX+1) é 
verdadeira.

Date: Mon, 10 Oct 2011 22:46:50 -0300
Subject: [obm-l] Re: [obm-l] fatoração de polinômio
From: pedromn...@gmail.com
To: obm-l@mat.puc-rio.br

sempre tem o wolfram alpha, http://www.wolframalpha.com/input/?i=+X^5%2BX%2B1+ 
, mas nao sei se eh esse o objetivo

Em 10 de outubro de 2011 21:57, Luan Gabriel luan_gabrie...@hotmail.com 
escreveu:






Boa noite, entrei hoje na lista,espero ter mandado pro e-mail certo. A questão 
é encontrar uma fatoração para o polinômio: X^5+X+1 
Agradeço a ajuda. 

  

[obm-l] FW: [obm-l] Re: [obm-l] fatoração de polinômio

2011-10-10 Por tôpico Luan Gabriel

Como falei, consegui provar pelo lema de gauss, substituindo x por x+1, que o 
polinômio é redutível nos Z, e assim aquele método de supor a fatoração fica 
restrito a encontrar inteiros que satisfaçam o problema.Mesmo assim, é um 
método muito braçal, acho que existe algo por trás do problema. Se alguém tiver 
uma luz, agradeço!

From: luan_gabrie...@hotmail.com
To: obm-l@mat.puc-rio.br
Subject: RE: [obm-l] Re: [obm-l] fatoração de polinômio
Date: Tue, 11 Oct 2011 05:17:33 +0300








Olhei o site, e realmente é muito bom. Quanto ao problema, ele não apresenta 
uma maneira prática de fatoração;pelo contrário, usa algo muito bizarro. De 
qualquer forma, vi a forma fatorada e,como era de se esperar, ele é redutível 
nos Z e a fatoração resulta em dois polinômios primitivos. Tentei provar que o 
polinômio inicial era redutível nos Z,mas não consegui. Então,não sei se a 
suposição de que o polinômio pode ser fatorado em (X^3+aX^2+bX+1).(X^2+cX+1) é 
verdadeira.

Date: Mon, 10 Oct 2011 22:46:50 -0300
Subject: [obm-l] Re: [obm-l] fatoração de polinômio
From: pedromn...@gmail.com
To: obm-l@mat.puc-rio.br

sempre tem o wolfram alpha, http://www.wolframalpha.com/input/?i=+X^5%2BX%2B1+ 
, mas nao sei se eh esse o objetivo

Em 10 de outubro de 2011 21:57, Luan Gabriel luan_gabrie...@hotmail.com 
escreveu:






Boa noite, entrei hoje na lista,espero ter mandado pro e-mail certo. A questão 
é encontrar uma fatoração para o polinômio: X^5+X+1 
Agradeço a ajuda. 


  

[obm-l] Re: [obm-l] fatoração de polinômio

2011-10-10 Por tôpico Luan Gabriel









Como falei, consegui provar pelo lema de gauss, substituindo x por x+1, que o 
polinômio é redutível nos Z, e assim aquele método de supor a fatoração fica 
restrito a encontrar inteiros que satisfaçam o problema.Mesmo assim, é um 
método muito braçal, acho que existe algo por trás do problema. Se alguém tiver 
uma luz, agradeço!



  

[obm-l] RE: [obm-l] RE: [obm-l] Re: [obm-l] fatoração de polinômio

2011-10-10 Por tôpico Luan Gabriel

Como falei, consegui provar pelo lema de gauss, substituindo x por x+1, que o 
polinômio é redutível nos Z, e assim aquele método de supor a fatoração fica 
restrito a encontrar inteiros que satisfaçam o problema.Mesmo assim, é um 
método muito braçal, acho que existe algo por trás do problema. Se alguém tiver 
uma luz, agradeço!
  


  

[obm-l] RE: [obm-l] fatoração de polinômio

2011-10-10 Por tôpico João Maldonado


Eu faria assim,   
x^5+ x  + 1 = T=x^5 + x^3 + x  + (1-x^3)=x(x^4 + x² + 1) + (1-x)(x²+x+1)
Fazendo k =(x^4 + x² + 1)y=x²temos  k=y²+y+1 =   
(y³-1)/(y-1)=(x^6-1)/(x²-1)=(x³-1)(x³+1)/(x+1)(x-1)=(x²-x+1)(x²+x+1)Logo   
T=(x²+x+1)(x³-x²+x)  + (1-x)(x²+x+1)T=(x²+x+1)(x³-x²+1)
[]'sJoão
From: luan_gabrie...@hotmail.com
To: obm-l@mat.puc-rio.br
Subject: [obm-l] fatoração de polinômio
Date: Tue, 11 Oct 2011 03:57:55 +0300








Boa noite, entrei hoje na lista,espero ter mandado pro e-mail certo. A questão 
é encontrar uma fatoração para o polinômio: X^5+X+1 
Agradeço a ajuda.   
  

[obm-l] Re: [obm-l] RE: [obm-l] RE: [obm-l] Re: [obm-l] RE: [obm-l] Fatoração

2010-06-21 Por tôpico Bernardo Freitas Paulo da Costa
2010/6/21 Thiago Tarraf Varella thiago_...@hotmail.com:
 Verdade!
 2(x+1)(x-1/2)(2x²-x+1)
 2(x+1)(2x-1)(2x²-x+1)/2
 (x+1)(2x-1)(2x²-x+1)
 Aí acaba, né?
Porquê ?

(2x^2 - x + 1) = (x - 1/4 - i*raiz(7)/4)*(x - 1/4 + i*raiz(7)/4)

Repare que dizer que não vale complexos é exatamente a mesma coisa
que dizer que também não vale fazer (x^3 - 2) = (x - raiz3(2))*(x^2 +
x*raiz3(2) + raiz3(4)). O que é mais ou menos arbitrário, a menos que
você especifique que você só aceita polinômios com coeficientes
racionais. E porquê não inteiros? Mas nada disso tá dito no enunciado.
Não que eu tenha visto muitos enunciados que digam explicitamente
fatore o polinômio abaixo em Q[X] (ou Z[X], ou R[X], ou ), mas
de certa forma isso é um equivalente do que o Ralph diz em
combinatória, aqui fica bom, o enunciado é vago, vamos supor que ele
quer dizer que é para fatorar em Z[X], que é o que parece razoável, já
que todos os coeficientes são inteiros.

 ;D

-- 
Bernardo Freitas Paulo da Costa

=
Instruções para entrar na lista, sair da lista e usar a lista em
http://www.mat.puc-rio.br/~obmlistas/obm-l.html
=


[obm-l] Fatoração

2010-06-20 Por tôpico Lucas Hagemaister

 

Como fatorar:

 

4x^4(x na quarta) -x² +2x -1

 

Tentei de várias maneiras, mas nunca consegui completar a fatoração.

 

Agradeço desde já.

 

Abraço
  
_
VEJA SEUS EMAILS ONDE QUER QUE VOCÊ ESTEJA, ACESSE O HOTMAIL PELO SEU CELULAR 
AGORA.
http://celular.windowslive.com.br/hotmail.asp?produto=Hotmailutm_source=Live_Hotmailutm_medium=Taglineutm_content=VEJASEUSEM84utm_campaign=MobileServices

[obm-l] Re: [obm-l] Fatoração

2010-06-20 Por tôpico Paulo Vedana
errata:  (2x²)² - (x-1)²

2010/6/20 Paulo Vedana paulo.ved...@poli.usp.br

 (2x)² - (x-1)²
 Agora é só fazer a diferença de quadrados e terminar.

 Dica: fatoração é pura PRÁTICA. Então, vai em frente que esse é o caminho!

 Abraço,

 Paulo Vedana.

 2010/6/20 Lucas Hagemaister lucashagemais...@msn.com


 Como fatorar:

 4x^4(x na quarta) -x² +2x -1

 Tentei de várias maneiras, mas nunca consegui completar a fatoração.

 Agradeço desde já.

 Abraço

 --
 O SEU NAVEGADOR PODE TE PROTEGER DE FRAUDES NA WEB. VEJA DICAS DE
 INTERNET EXPLORER 
 8http://www.microsoft.com/brasil/windows/internet-explorer/features/stay-safer-online.aspx?tabid=1catid=1WT.mc_id=1588





[obm-l] Re: [obm-l] RE: [obm-l] Fatoração

2010-06-20 Por tôpico Paulo Vedana
ainda dá pra fatorar mais!

2010/6/20 Thiago Tarraf Varella thiago_...@hotmail.com

  Eu cheguei nisso:
 4x^4 - x² + 2x - 1
 4x^4 - (x²-2x+1) 3o./4o. Caso de fatoração:
 4x^4 - (x-1)²
 (2x²)² - (x-1)² 4o./5o. Caso de fatoração:
 (2x² + x - 1)(2x² - x + 1)
 Espero que tenha ajudado!
 Thiago



 --
 From: lucashagemais...@msn.com
 To: obm-l@mat.puc-rio.br
 Subject: [obm-l] Fatoração
 Date: Sun, 20 Jun 2010 20:01:09 -0300



 Como fatorar:

 4x^4(x na quarta) -x² +2x -1

 Tentei de várias maneiras, mas nunca consegui completar a fatoração.

 Agradeço desde já.

 Abraço

 --
 O SEU NAVEGADOR PODE TE PROTEGER DE FRAUDES NA WEB. VEJA DICAS DE INTERNET
 EXPLORER 
 8http://www.microsoft.com/brasil/windows/internet-explorer/features/stay-safer-online.aspx?tabid=1catid=1WT.mc_id=1588

 --
 TRANSFORME SUAS FOTOS EM EMOTICONS PARA O MESSENGER. CLIQUE AQUI E VEJA
 COMO.http://ilm.windowslive.com.br/?ocid=ILM:Live:Hotmail:Tagline:1x1:TRANSFORME77:-



[obm-l] RE: [obm-l] RE: [obm -l] Re: [obm-l] RE: [obm-l] Fatoração

2010-06-20 Por tôpico Thiago Tarraf Varella

Verdade!2(x+1)(x-1/2)(2x²-x+1)2(x+1)(2x-1)(2x²-x+1)/2(x+1)(2x-1)(2x²-x+1)Aí 
acaba, né?;D
From: lucashagemais...@msn.com
To: obm-l@mat.puc-rio.br
Subject: [obm-l] RE: [obm-l] Re: [obm-l] RE: [obm-l] Fatoração
Date: Sun, 20 Jun 2010 22:44:44 -0300








Esquece, entendi o pq. Obrigado =)
 


Date: Sun, 20 Jun 2010 22:20:30 -0300
Subject: [obm-l] Re: [obm-l] RE: [obm-l] Fatoração
From: paulo.ved...@poli.usp.br
To: obm-l@mat.puc-rio.br

ainda dá pra fatorar mais!


2010/6/20 Thiago Tarraf Varella thiago_...@hotmail.com



Eu cheguei nisso:
4x^4 - x² + 2x - 1
4x^4 - (x²-2x+1) 3o./4o. Caso de fatoração:
4x^4 - (x-1)²
(2x²)² - (x-1)² 4o./5o. Caso de fatoração:
(2x² + x - 1)(2x² - x + 1)
Espero que tenha ajudado!
Thiago







From: lucashagemais...@msn.com
To: obm-l@mat.puc-rio.br
Subject: [obm-l] Fatoração
Date: Sun, 20 Jun 2010 20:01:09 -0300




 
Como fatorar:
 
4x^4(x na quarta) -x² +2x -1
 
Tentei de várias maneiras, mas nunca consegui completar a fatoração.
 
Agradeço desde já.
 
Abraço



O SEU NAVEGADOR PODE TE PROTEGER DE FRAUDES NA WEB. VEJA DICAS DE INTERNET 
EXPLORER 8


TRANSFORME SUAS FOTOS EM EMOTICONS PARA O MESSENGER. CLIQUE AQUI E VEJA COMO.
  
O SEU NAVEGADOR PODE TE PROTEGER DE FRAUDES NA WEB. VEJA DICAS DE INTERNET 
EXPLORER 8 
_
ACESSE O MESSENGER DO SEU CELULAR AGORA MESMO. CLIQUE E VEJA AQUI UM PASSO A 
PASSO.
http://celular.windowslive.com.br/messenger.asp?produto=Messengerutm_source=Live_Hotmailutm_medium=Taglineutm_content=ACESSEOMES83utm_campaign=MobileServices

[obm-l] Re: [obm-l] RES: [obm-l] Fatoração Básica

2009-05-03 Por tôpico Denisson
Bem, vc pode considerar que a^2 - b^2 é um polinômio do segundo grau em que
a é a variável e -b^2 é constante dada e fatorá-lo.

Para fatorá-lo você pode aplicar a fórmula de resoluç]ão da equação do
segundo grau e obter b ou -b como raizes, logo fatoramos
a^2 - b^2 = (a + b)(a - b)

No caso de a^3 - b^3 use o mesmo artifício, observe que obviamente a=b é uma
solução da equação. Logo a^3 - b^3 é divisivel por (a-b) efetuando essa
divisão obtemos a^3 - b^3 = (a-b)(a^2 + ab + b^2)

Isso vale pra a^3 + b^3 e a^2 + b^2. Na verdade dá pra fazer isso pra
qualquer caso, mas a resolução vai se tornando mais dificil.

Mas é como o colega falou, à medida que a matemática vai virando uma
ferramenta constante na sua vida essas relações viram naturais :P

2009/5/2 Jayro Bedoff barz...@dglnet.com.br

  Caro Hugleo é claro que podemos deduzir essas fórmulas com pequenos
 truques de álgebra básica ( aliás é um bom exercício ), todavia essas
 expressões são utilizadas tantas vezes na matemática ( elementar ou não )
 que sabe-las de cor é uma necessidade! No caso 1 experimente somar e
 subtrair ab da diferença de quadrados.

 Um abraço.

 Jayro Bedoff


  --

 *De:* owner-ob...@mat.puc-rio.br [mailto:owner-ob...@mat.puc-rio.br] *Em
 nome de *HugLeo
 *Enviada em:* sábado, 2 de maio de 2009 01:32
 *Para:* obm-l@mat.puc-rio.br
 *Assunto:* [obm-l] Fatoração Básica



 Algumas vezes temos necessidade de fatorar uma expressão para resolver um
 problema maior.
 Seja por exemplo as seguintes:

 1) a^2 - b^2 = (a - b)(a + b)
 2) a^3 - b^3 = (a - b)(a^2 + ab + b^2)

 Usando a propriedade distributiva você pode facilmente expandir a expressão
 do lado direito e chegar à do lado esquerdo.
 Mas quando necessitamos sair da experessão do lado esquerdo para chegar na
 expressão fatorado do lado direito fica mais complicado.
 Essas são fórmulas básicas da diferença de quadrados e diferença de cubos
 respectivamente. Elas podem ajudar a simplificar outras expressões.
 Entretando, devido elas não serem usadas sempre em determinados problemas
 acabamos por esquecê-las.
 Então, como deduzi-las na hora sem a necessidade de decorá-las?

 --
 -hUgLeO-♑




-- 
Denisson


[obm-l] Fatoração Básica

2009-05-02 Por tôpico HugLeo
Algumas vezes temos necessidade de fatorar uma expressão para resolver um
problema maior.
Seja por exemplo as seguintes:

1) a^2 - b^2 = (a - b)(a + b)
2) a^3 - b^3 = (a - b)(a^2 + ab + b^2)

Usando a propriedade distributiva você pode facilmente expandir a expressão
do lado direito e chegar à do lado esquerdo.
Mas quando necessitamos sair da experessão do lado esquerdo para chegar na
expressão fatorado do lado direito fica mais complicado.
Essas são fórmulas básicas da diferença de quadrados e diferença de cubos
respectivamente. Elas podem ajudar a simplificar outras expressões.
Entretando, devido elas não serem usadas sempre em determinados problemas
acabamos por esquecê-las.
Então, como deduzi-las na hora sem a necessidade de decorá-las?

-- 
-hUgLeO-♑


[obm-l] RES: [obm-l] Fatoração Básica

2009-05-02 Por tôpico Jayro Bedoff
Caro Hugleo é claro que podemos deduzir essas fórmulas com pequenos truques de 
álgebra básica ( aliás é um bom exercício ), todavia essas expressões são 
utilizadas tantas vezes na matemática ( elementar ou não ) que sabe-las de cor 
é uma necessidade! No caso 1 experimente somar e subtrair ab da diferença de 
quadrados.

Um abraço.

Jayro Bedoff

 

  _  

De: owner-ob...@mat.puc-rio.br [mailto:owner-ob...@mat.puc-rio.br] Em nome de 
HugLeo
Enviada em: sábado, 2 de maio de 2009 01:32
Para: obm-l@mat.puc-rio.br
Assunto: [obm-l] Fatoração Básica

 

Algumas vezes temos necessidade de fatorar uma expressão para resolver um 
problema maior.
Seja por exemplo as seguintes:

1) a^2 - b^2 = (a - b)(a + b)
2) a^3 - b^3 = (a - b)(a^2 + ab + b^2)

Usando a propriedade distributiva você pode facilmente expandir a expressão do 
lado direito e chegar à do lado esquerdo.
Mas quando necessitamos sair da experessão do lado esquerdo para chegar na 
expressão fatorado do lado direito fica mais complicado.
Essas são fórmulas básicas da diferença de quadrados e diferença de cubos 
respectivamente. Elas podem ajudar a simplificar outras expressões. Entretando, 
devido elas não serem usadas sempre em determinados problemas acabamos por 
esquecê-las.
Então, como deduzi-las na hora sem a necessidade de decorá-las?

-- 
-hUgLeO-♑



[obm-l] Re: [obm-l] Re: [obm-l] Re: [obm-l] Fatoração de 5 ^1985 - 1.

2009-04-15 Por tôpico Carlos Alberto da Silva Victor
Olá ,
Esta  questão realmente não é fácil , como de repente pode  parecer . Ela
foi  proposta numa  Olimpíada  Internacional e não usada e, foi também
 proposta na  RPM - 18 . A solução do Vidal  teve um brilhantismo , pois
explicou  em detalhes os passos .

Abraços

Carlos  Victor

2009/4/6 Carlos Nehab ne...@infolink.com.br

 Oi, Vidal (e Fabricio),

 Já que meu neto não está aqui em casa... :-)   e como gostei tanto de suas
 continhas de cabeça, fucei um site que tenho certeza que vocês vão
 gostar Tem coisas surreais
 http://www.leyland.vispa.com/numth/factorization/main.htm

 Abraços,
 Nehab
 (
 *Vidal escreveu:

 Caro Fabrício,

 Eu também passei por esta etapa (produto de dois polinômios de grau 2)
 durante o pequeno tempo que pensei na solução, depois de provocado pelo
 Nehab. Mas infelizmente os fatores não eram inteiros.

 Abraços,
 Vidal.

 :: vi...@mail.com



 2009/4/6 fabrici...@usp.br fabrici...@usp.br

 Vidal, muito boa a sacada.

 Eu tinha tentado escrever como o produto de dois polinômios de grau 2, sem
 sucesso.
 Parabéns pela solução.

 Um abraço.

 .


 On Apr 6, 2009, at 03:21 , *Vidal wrote:

 Caros Fabrício e Nehab,

 Achar um fator foi fácil, o problema foi quebrar o quociente nos outros
 dois.

 Fiz assim:

 5^1985 - 1 = (5^397)^5 - 1

 Seja x = 5^397.

 Então queremos fatorar x^5 - 1 que, de imediato, resulta em (x - 1) (x^4
  + x^3 + x^2 + x + 1), ou seja, um dos fatores é 5^397 - 1.

 Falta fatorar x^4  + x^3 + x^2 + x + 1 de uma forma conveniente.

 Após um tempinho (pouca coisa, até no Fla x Flu no Maracanã estava
 rabiscando...), tive a idéia de tentar escrever a expressão como uma
 adequada diferença de dois quadrados. Caso conseguisse, o problema estaria
 resolvido, pois um fator seria a soma e outro, a diferença.

 Arbitrei o primeiro quadrado como (x^2 + ax + 1)^2, que já geraria o
 termo de quarto grau e o termo independente corretos.

 E coloquei o segundo quadrado como 5x(x+b)^2, pois como x = 5^397, 5x =
 5^398 seria um quadrado perfeito.

 Igualando as expressões (e rezando para encontrar valores de a e b
 compatíveis), veio:

 (x^2 + ax + 1)^2 - 5x(x+b)^2 = x^4 + (2a -5)x^3 + (a^2 - 10b + 2)x^2 +
 (2a - 5b^2)x + 1 = x^4  + x^3 + x^2 + x + 1

 Assim:

 2a -5 = 1 = a = 3
 a^2 - 10b + 2 = 1 = b = 1

 Agora era hora da onça beber água:

 2a - 5b^2 = 1

 Mas a = 3 e b = 1 satisfazem ! Eureka !

 x^4  + x^3 + x^2 + x + 1 = (x^2 + 3x + 1)^2 - 5x(x+1)^2

 Substituindo x por 5^397:

 ((5^397)^2 + 3*5^397 +1)^2 - 5*5^397*(5^397 + 1)^2 =

 = ((5^397)^2 + 3*5^397 +1)^2 - 5^398*(5^397 + 1)^2 (diferença de
 quadrados) =

 = (((5^397)^2 + 3*5^397 +1) - 5^199*(5^397 + 1)) * (((5^397)^2 + 3*5^397
 +1) + 5^199*(5^397 + 1)) (produto da diferença pela soma) =

 = (5^794 - 5^596 + 3*5^397 - 5^199 + 1) * (5^794 + 5^596 + 3*5^397 +
 5^199 + 1)

 Os três fatores são claramente maiores que 5^100, conforme solicitado.

 Então:

 5^1985 -1 = (5^397 - 1) * (5^794 - 5^596 + 3*5^397 - 5^199 + 1) * (5^794
 + 5^596 + 3*5^397 + 5^199 + 1)

 Como já são três da manhã e já perdi o sono mesmo, resolvi fazer umas
 continhas de cabeça, tal como o Ralph fez outro dia desses...

 5^397-1 = 2 x 2 x 1.043.801.929 x 7.768.438.039 x C258

 5^794 -5^596 + 3*5^397 - 5^199 + 1 = 71 x 399.091.951.801 x C542

 5^794 + 5^596 + 3*5^397 + 5^199 + 1 = 11 x 146.891 x C549

 Logo:

 5^1985 -1 = 2 x 2 x 11 x 71 x 146.891 x 1.043.801.929 x 7.768.438.039 x
 399.091.951.801 x C258 x C542 x C549

 (onde Cn são números compostos de n algarismos).

 A fatoração de C258, C542 e C549 fica como exercício ...

 :)

 Abraços,
 Vidal.

 P.S. Nehab: Apesar de não nos conhecermos pessoalmente, temos um grande
 amigo em comum: o Manuel Martins Filho, professor de Informática da Carioca
 ! Abraços !

 :: vi...@mail.com

 ***

 2009/4/5 Carlos Nehab ne...@infolink.com.br
 Oi, gente,

 Fabricio postou este interessante problema e aparentemente ninguém deu
 muita bola, talvez achando que é óbvio.
 Não achei óbvio não.  Quem resolveu?

 Abraços,
 Nehab

 fabrici...@usp.br escreveu:

 Caros colegas,

 mexendo em algumas listas antigas de exercícios, um me chamou muito a
 atenção.
 Pede pra fatorar 5^1985 - 1 num produto de três inteiros maiores que
 5^100.

 Pra facilitar um possível avanço, 1985 pode ser escrito como 5 x 397
 (ambos primos).

 .

 =
 Instruções para entrar na lista, sair da lista e usar a lista em
 http://www.mat.puc-rio.br/~obmlistas/obm-l.html

 =

 =
 Instruções para entrar na lista, sair da lista e usar a lista em
 http://www.mat.puc-rio.br/~obmlistas/obm-l.html=



 =
 Instruções para entrar na lista, sair da lista e usar a lista em
 

Re: [obm-l] Re: [obm-l] Fatoração de 5^ 1985 - 1.

2009-04-15 Por tôpico Carlos Nehab




Oi, Vidal,

Muito legal a sacao bem sucedida de forar a diferena entre
quadrados, e com muita criatividade ... Eu no tinha conseguido matar
o problema.

Quanto ao Manuel somos amigos h 30 anos e j percorremos muito cho
juntos. Nos conhecemos no SERPRO, quando ramos funcionrios de uma
rea maluca de Estatstica, Modelagem , etc (era onde eles colocavam
os caras que, alm de programar, como todo mundo de l programava,
sabiam tambm fazer umas continhas mgicas como a que voc fez no
problema abaixo..). E nesta poca eu ainda dava aula no IME, de
Lgica, Anlise Linear, Clculo ,1,2..., N..., etc). Pr voc ter uma
idia meu cargo era de Matemgico 

Ahhh , que emoo quando penso nas pessoas
bacanas com quem convivi naquela poca. Todas geniais... Gostosas
saudades... 
Mas no sei se voc sabe, eu fui coordenador de Cursos de
Computao da Carioca e Gerente de Tecnologia durante uns 2 anos, h
uns 10 anos ... E l estava o Manuel que foi quem me seduziu a
trabalhar l...

Um grande abrao,
Nehab

PS: De onde voc conhece o Manuel? Da night? Dos botequins e rodadas de
violo? Ou foi aluno dele?

*Vidal escreveu:
Caros Fabrcio e Nehab,
  
Achar um fator foi fcil, o problema foi "quebrar" o quociente nos
outros dois.
  
Fiz assim:
  
5^1985 - 1 = (5^397)^5 - 1
  
Seja x = 5^397.
  
Ento queremos fatorar x^5 - 1 que, de imediato, resulta em (x - 1)
(x^4 + x^3 + x^2 + x + 1), ou seja, um dos fatores  5^397 - 1.
  
Falta fatorar x^4 + x^3 + x^2 + x + 1 de uma forma conveniente.
  
Aps um tempinho (pouca coisa, at no Fla x Flu no Maracan estava
rabiscando...), tive a idia de tentar escrever a expresso como uma
adequada diferena de dois quadrados. Caso conseguisse, o problema
estaria resolvido, pois um fator seria a soma e outro, a diferena.
  
Arbitrei o primeiro quadrado como (x^2 + ax + 1)^2, que j geraria o
termo de quarto grau e o termo independente corretos.
  
E coloquei o segundo quadrado como 5x(x+b)^2, pois como x = 5^397, 5x =
5^398 seria um quadrado perfeito.
  
Igualando as expresses (e rezando para encontrar valores de a e b
compatveis), veio:
  
(x^2 + ax + 1)^2 - 5x(x+b)^2 = x^4 + (2a -5)x^3 + (a^2 - 10b + 2)x^2 +
(2a - 5b^2)x + 1 = x^4 + x^3 + x^2 + x + 1
  
Assim:
  
2a -5 = 1 = a = 3
a^2 - 10b + 2 = 1 = b = 1
  
Agora era hora da ona beber gua:
  
2a - 5b^2 = 1
  
Mas a = 3 e b = 1 satisfazem ! Eureka !
  
x^4 + x^3 + x^2 + x + 1 = (x^2 + 3x + 1)^2 - 5x(x+1)^2
  
Substituindo x por 5^397:
  
((5^397)^2 + 3*5^397 +1)^2 - 5*5^397*(5^397 + 1)^2 =
  
= ((5^397)^2 + 3*5^397 +1)^2 - 5^398*(5^397 + 1)^2 (diferena de
quadrados) =
  
= (((5^397)^2 + 3*5^397 +1) - 5^199*(5^397 + 1)) * (((5^397)^2 +
3*5^397 +1) + 5^199*(5^397 + 1)) (produto da diferena pela soma) =
  
= (5^794 - 5^596 + 3*5^397 - 5^199 + 1) * (5^794 + 5^596 + 3*5^397 +
5^199 + 1)
  
Os trs fatores so claramente maiores que 5^100, conforme solicitado.
  
Ento:
  
5^1985 -1 = (5^397 - 1) * (5^794 - 5^596 + 3*5^397 - 5^199 + 1) *
(5^794 + 5^596 + 3*5^397 + 5^199 + 1)
  
Como j so trs da manh e j perdi o sono mesmo, resolvi fazer umas
"continhas de cabea", tal como o Ralph fez outro dia desses...
  
5^397-1 = 2 x 2 x 1.043.801.929 x 7.768.438.039 x C258
  
5^794 -5^596 + 3*5^397 - 5^199 + 1 = 71 x 399.091.951.801 x C542
  
5^794 + 5^596 + 3*5^397 + 5^199 + 1 = 11 x 146.891 x C549
  
Logo:
  
5^1985 -1 = 2 x 2 x 11 x 71 x 146.891 x 1.043.801.929 x 7.768.438.039 x
399.091.951.801 x C258 x C542 x C549
  
(onde Cn so nmeros compostos de n algarismos).
  
A fatorao de C258, C542 e C549 fica como exerccio ...
  
:)
  
Abraos,
Vidal.
  
P.S. Nehab: Apesar de no nos conhecermos pessoalmente, temos um grande
amigo em comum: o Manuel Martins Filho, professor de Informtica da
Carioca ! Abraos !
  
:: vi...@mail.com
  
***
  
  2009/4/5 Carlos Nehab ne...@infolink.com.br
  
Oi, gente,

Fabricio postou este interessante problema e aparentemente ningum deu
muita bola, talvez achando que  bvio. 
No achei bvio no. Quem resolveu? 

Abraos, 
Nehab

fabrici...@usp.br escreveu:


Caros colegas, 
  
mexendo em algumas listas antigas de exerccios, um me chamou muito a
ateno. 
Pede pra fatorar 5^1985 - 1 num produto de trs inteiros maiores que
5^100. 
  
Pra facilitar um possvel avano, 1985 pode ser escrito como 5 x 397
(ambos primos). 
  
. 
=
  
Instrues para entrar na lista, sair da lista e usar a lista em 
  http://www.mat.puc-rio.br/~obmlistas/obm-l.html 
=
  
  




=
Instrues para entrar na lista, sair da lista e usar a lista em
http://www.mat.puc-rio.br/~obmlistas/obm-l.html
=
  
  
  



=
Instruções para entrar na lista, sair da lista e usar a 

[obm-l] Re: [obm-l] Fatoração de 5^1985 - 1.

2009-04-06 Por tôpico *Vidal
Caros Fabrício e Nehab,

Achar um fator foi fácil, o problema foi quebrar o quociente nos outros
dois.

Fiz assim:

5^1985 - 1 = (5^397)^5 - 1

Seja x = 5^397.

Então queremos fatorar x^5 - 1 que, de imediato, resulta em (x - 1) (x^4  +
x^3 + x^2 + x + 1), ou seja, um dos fatores é 5^397 - 1.

Falta fatorar x^4  + x^3 + x^2 + x + 1 de uma forma conveniente.

Após um tempinho (pouca coisa, até no Fla x Flu no Maracanã estava
rabiscando...), tive a idéia de tentar escrever a expressão como uma
adequada diferença de dois quadrados. Caso conseguisse, o problema estaria
resolvido, pois um fator seria a soma e outro, a diferença.

Arbitrei o primeiro quadrado como (x^2 + ax + 1)^2, que já geraria o termo
de quarto grau e o termo independente corretos.

E coloquei o segundo quadrado como 5x(x+b)^2, pois como x = 5^397, 5x =
5^398 seria um quadrado perfeito.

Igualando as expressões (e rezando para encontrar valores de a e b
compatíveis), veio:

(x^2 + ax + 1)^2 - 5x(x+b)^2 = x^4 + (2a -5)x^3 + (a^2 - 10b + 2)x^2 + (2a -
5b^2)x + 1 = x^4  + x^3 + x^2 + x + 1

Assim:

2a -5 = 1 = a = 3
a^2 - 10b + 2 = 1 = b = 1

Agora era hora da onça beber água:

2a - 5b^2 = 1

Mas a = 3 e b = 1 satisfazem ! Eureka !

x^4  + x^3 + x^2 + x + 1 = (x^2 + 3x + 1)^2 - 5x(x+1)^2

Substituindo x por 5^397:

((5^397)^2 + 3*5^397 +1)^2 - 5*5^397*(5^397 + 1)^2 =

= ((5^397)^2 + 3*5^397 +1)^2 - 5^398*(5^397 + 1)^2 (diferença de quadrados)
=

= (((5^397)^2 + 3*5^397 +1) - 5^199*(5^397 + 1)) * (((5^397)^2 + 3*5^397 +1)
+ 5^199*(5^397 + 1)) (produto da diferença pela soma) =

= (5^794 - 5^596 + 3*5^397 - 5^199 + 1) * (5^794 + 5^596 + 3*5^397 + 5^199 +
1)

Os três fatores são claramente maiores que 5^100, conforme solicitado.

Então:

5^1985 -1 = (5^397 - 1) * (5^794 - 5^596 + 3*5^397 - 5^199 + 1) * (5^794 +
5^596 + 3*5^397 + 5^199 + 1)

Como já são três da manhã e já perdi o sono mesmo, resolvi fazer umas
continhas de cabeça, tal como o Ralph fez outro dia desses...

5^397-1 = 2 x 2 x 1.043.801.929 x 7.768.438.039 x C258

5^794 -5^596 + 3*5^397 - 5^199 + 1 = 71 x 399.091.951.801 x C542

5^794 + 5^596 + 3*5^397 + 5^199 + 1 = 11 x 146.891 x C549

Logo:

5^1985 -1 = 2 x 2 x 11 x 71 x 146.891 x 1.043.801.929 x 7.768.438.039 x
399.091.951.801 x C258 x C542 x C549

(onde Cn são números compostos de n algarismos).

A fatoração de C258, C542 e C549 fica como exercício ...

:)

Abraços,
Vidal.

P.S. Nehab: Apesar de não nos conhecermos pessoalmente, temos um grande
amigo em comum: o Manuel Martins Filho, professor de Informática da Carioca
! Abraços !

:: vi...@mail.com

***

2009/4/5 Carlos Nehab ne...@infolink.com.br

  Oi, gente,

 Fabricio postou este interessante problema e aparentemente ninguém deu
 muita bola, talvez achando que é óbvio.
 Não achei óbvio não.  Quem resolveu?

 Abraços,
 Nehab

 fabrici...@usp.br escreveu:

 Caros colegas,

 mexendo em algumas listas antigas de exercícios, um me chamou muito a
 atenção.
 Pede pra fatorar 5^1985 - 1 num produto de três inteiros maiores que 5^100.


 Pra facilitar um possível avanço, 1985 pode ser escrito como 5 x 397 (ambos
 primos).

 .
 =
 Instruções para entrar na lista, sair da lista e usar a lista em
 http://www.mat.puc-rio.br/~obmlistas/obm-l.htmlhttp://www.mat.puc-rio.br/%7Eobmlistas/obm-l.html
 =

  =
 Instruções para entrar na lista, sair da lista e usar a lista em
 http://www.mat.puc-rio.br/~obmlistas/obm-l.htmlhttp://www.mat.puc-rio.br/%7Eobmlistas/obm-l.html=


Re: [obm-l] Re: [obm-l] Fatoração de 5^1985 - 1.

2009-04-06 Por tôpico fabrici...@usp.br

Vidal, muito boa a sacada.

Eu tinha tentado escrever como o produto de dois polinômios de grau  
2, sem sucesso.

Parabéns pela solução.

Um abraço.

.


On Apr 6, 2009, at 03:21 , *Vidal wrote:


Caros Fabrício e Nehab,

Achar um fator foi fácil, o problema foi quebrar o quociente nos  
outros dois.


Fiz assim:

5^1985 - 1 = (5^397)^5 - 1

Seja x = 5^397.

Então queremos fatorar x^5 - 1 que, de imediato, resulta em (x - 1)  
(x^4  + x^3 + x^2 + x + 1), ou seja, um dos fatores é 5^397 - 1.


Falta fatorar x^4  + x^3 + x^2 + x + 1 de uma forma conveniente.

Após um tempinho (pouca coisa, até no Fla x Flu no Maracanã estava  
rabiscando...), tive a idéia de tentar escrever a expressão como  
uma adequada diferença de dois quadrados. Caso conseguisse, o  
problema estaria resolvido, pois um fator seria a soma e outro, a  
diferença.


Arbitrei o primeiro quadrado como (x^2 + ax + 1)^2, que já geraria  
o termo de quarto grau e o termo independente corretos.


E coloquei o segundo quadrado como 5x(x+b)^2, pois como x = 5^397,  
5x = 5^398 seria um quadrado perfeito.


Igualando as expressões (e rezando para encontrar valores de a e b  
compatíveis), veio:


(x^2 + ax + 1)^2 - 5x(x+b)^2 = x^4 + (2a -5)x^3 + (a^2 - 10b + 2) 
x^2 + (2a - 5b^2)x + 1 = x^4  + x^3 + x^2 + x + 1


Assim:

2a -5 = 1 = a = 3
a^2 - 10b + 2 = 1 = b = 1

Agora era hora da onça beber água:

2a - 5b^2 = 1

Mas a = 3 e b = 1 satisfazem ! Eureka !

x^4  + x^3 + x^2 + x + 1 = (x^2 + 3x + 1)^2 - 5x(x+1)^2

Substituindo x por 5^397:

((5^397)^2 + 3*5^397 +1)^2 - 5*5^397*(5^397 + 1)^2 =

= ((5^397)^2 + 3*5^397 +1)^2 - 5^398*(5^397 + 1)^2 (diferença de  
quadrados) =


= (((5^397)^2 + 3*5^397 +1) - 5^199*(5^397 + 1)) * (((5^397)^2 +  
3*5^397 +1) + 5^199*(5^397 + 1)) (produto da diferença pela soma) =


= (5^794 - 5^596 + 3*5^397 - 5^199 + 1) * (5^794 + 5^596 + 3*5^397  
+ 5^199 + 1)


Os três fatores são claramente maiores que 5^100, conforme solicitado.

Então:

5^1985 -1 = (5^397 - 1) * (5^794 - 5^596 + 3*5^397 - 5^199 + 1) *  
(5^794 + 5^596 + 3*5^397 + 5^199 + 1)


Como já são três da manhã e já perdi o sono mesmo, resolvi fazer  
umas continhas de cabeça, tal como o Ralph fez outro dia desses...


5^397-1 = 2 x 2 x 1.043.801.929 x 7.768.438.039 x C258

5^794 -5^596 + 3*5^397 - 5^199 + 1 = 71 x 399.091.951.801 x C542

5^794 + 5^596 + 3*5^397 + 5^199 + 1 = 11 x 146.891 x C549

Logo:

5^1985 -1 = 2 x 2 x 11 x 71 x 146.891 x 1.043.801.929 x  
7.768.438.039 x 399.091.951.801 x C258 x C542 x C549


(onde Cn são números compostos de n algarismos).

A fatoração de C258, C542 e C549 fica como exercício ...

:)

Abraços,
Vidal.

P.S. Nehab: Apesar de não nos conhecermos pessoalmente, temos um  
grande amigo em comum: o Manuel Martins Filho, professor de  
Informática da Carioca ! Abraços !


:: vi...@mail.com

***

2009/4/5 Carlos Nehab ne...@infolink.com.br
Oi, gente,

Fabricio postou este interessante problema e aparentemente ninguém  
deu muita bola, talvez achando que é óbvio.

Não achei óbvio não.  Quem resolveu?

Abraços,
Nehab

fabrici...@usp.br escreveu:

Caros colegas,

mexendo em algumas listas antigas de exercícios, um me chamou  
muito a atenção.
Pede pra fatorar 5^1985 - 1 num produto de três inteiros maiores  
que 5^100.


Pra facilitar um possível avanço, 1985 pode ser escrito como 5 x  
397 (ambos primos).


.
= 


Instruções para entrar na lista, sair da lista e usar a lista em
http://www.mat.puc-rio.br/~obmlistas/obm-l.html
= 



== 
=== Instruções para entrar na lista, sair da lista e usar a lista  
em http://www.mat.puc-rio.br/~obmlistas/obm-l.html  
== 
===





=
Instruções para entrar na lista, sair da lista e usar a lista em
http://www.mat.puc-rio.br/~obmlistas/obm-l.html
=


[obm-l] Re: [obm-l] Re: [obm-l] Fatoração de 5^1985 - 1.

2009-04-06 Por tôpico *Vidal
Caro Fabrício,

Eu também passei por esta etapa (produto de dois polinômios de grau 2)
durante o pequeno tempo que pensei na solução, depois de provocado pelo
Nehab. Mas infelizmente os fatores não eram inteiros.

Abraços,
Vidal.

:: vi...@mail.com



2009/4/6 fabrici...@usp.br fabrici...@usp.br

 Vidal, muito boa a sacada.

 Eu tinha tentado escrever como o produto de dois polinômios de grau 2, sem
 sucesso.
 Parabéns pela solução.

 Um abraço.

 .



 On Apr 6, 2009, at 03:21 , *Vidal wrote:

  Caros Fabrício e Nehab,

 Achar um fator foi fácil, o problema foi quebrar o quociente nos outros
 dois.

 Fiz assim:

 5^1985 - 1 = (5^397)^5 - 1

 Seja x = 5^397.

 Então queremos fatorar x^5 - 1 que, de imediato, resulta em (x - 1) (x^4
  + x^3 + x^2 + x + 1), ou seja, um dos fatores é 5^397 - 1.

 Falta fatorar x^4  + x^3 + x^2 + x + 1 de uma forma conveniente.

 Após um tempinho (pouca coisa, até no Fla x Flu no Maracanã estava
 rabiscando...), tive a idéia de tentar escrever a expressão como uma
 adequada diferença de dois quadrados. Caso conseguisse, o problema estaria
 resolvido, pois um fator seria a soma e outro, a diferença.

 Arbitrei o primeiro quadrado como (x^2 + ax + 1)^2, que já geraria o termo
 de quarto grau e o termo independente corretos.

 E coloquei o segundo quadrado como 5x(x+b)^2, pois como x = 5^397, 5x =
 5^398 seria um quadrado perfeito.

 Igualando as expressões (e rezando para encontrar valores de a e b
 compatíveis), veio:

 (x^2 + ax + 1)^2 - 5x(x+b)^2 = x^4 + (2a -5)x^3 + (a^2 - 10b + 2)x^2 + (2a
 - 5b^2)x + 1 = x^4  + x^3 + x^2 + x + 1

 Assim:

 2a -5 = 1 = a = 3
 a^2 - 10b + 2 = 1 = b = 1

 Agora era hora da onça beber água:

 2a - 5b^2 = 1

 Mas a = 3 e b = 1 satisfazem ! Eureka !

 x^4  + x^3 + x^2 + x + 1 = (x^2 + 3x + 1)^2 - 5x(x+1)^2

 Substituindo x por 5^397:

 ((5^397)^2 + 3*5^397 +1)^2 - 5*5^397*(5^397 + 1)^2 =

 = ((5^397)^2 + 3*5^397 +1)^2 - 5^398*(5^397 + 1)^2 (diferença de
 quadrados) =

 = (((5^397)^2 + 3*5^397 +1) - 5^199*(5^397 + 1)) * (((5^397)^2 + 3*5^397
 +1) + 5^199*(5^397 + 1)) (produto da diferença pela soma) =

 = (5^794 - 5^596 + 3*5^397 - 5^199 + 1) * (5^794 + 5^596 + 3*5^397 + 5^199
 + 1)

 Os três fatores são claramente maiores que 5^100, conforme solicitado.

 Então:

 5^1985 -1 = (5^397 - 1) * (5^794 - 5^596 + 3*5^397 - 5^199 + 1) * (5^794 +
 5^596 + 3*5^397 + 5^199 + 1)

 Como já são três da manhã e já perdi o sono mesmo, resolvi fazer umas
 continhas de cabeça, tal como o Ralph fez outro dia desses...

 5^397-1 = 2 x 2 x 1.043.801.929 x 7.768.438.039 x C258

 5^794 -5^596 + 3*5^397 - 5^199 + 1 = 71 x 399.091.951.801 x C542

 5^794 + 5^596 + 3*5^397 + 5^199 + 1 = 11 x 146.891 x C549

 Logo:

 5^1985 -1 = 2 x 2 x 11 x 71 x 146.891 x 1.043.801.929 x 7.768.438.039 x
 399.091.951.801 x C258 x C542 x C549

 (onde Cn são números compostos de n algarismos).

 A fatoração de C258, C542 e C549 fica como exercício ...

 :)

 Abraços,
 Vidal.

 P.S. Nehab: Apesar de não nos conhecermos pessoalmente, temos um grande
 amigo em comum: o Manuel Martins Filho, professor de Informática da Carioca
 ! Abraços !

 :: vi...@mail.com

 ***

 2009/4/5 Carlos Nehab ne...@infolink.com.br
 Oi, gente,

 Fabricio postou este interessante problema e aparentemente ninguém deu
 muita bola, talvez achando que é óbvio.
 Não achei óbvio não.  Quem resolveu?

 Abraços,
 Nehab

 fabrici...@usp.br escreveu:

 Caros colegas,

 mexendo em algumas listas antigas de exercícios, um me chamou muito a
 atenção.
 Pede pra fatorar 5^1985 - 1 num produto de três inteiros maiores que
 5^100.

 Pra facilitar um possível avanço, 1985 pode ser escrito como 5 x 397
 (ambos primos).

 .
 =
 Instruções para entrar na lista, sair da lista e usar a lista em
 http://www.mat.puc-rio.br/~obmlistas/obm-l.htmlhttp://www.mat.puc-rio.br/%7Eobmlistas/obm-l.html
 =

 =
 Instruções para entrar na lista, sair da lista e usar a lista em
 http://www.mat.puc-rio.br/~obmlistas/obm-l.htmlhttp://www.mat.puc-rio.br/%7Eobmlistas/obm-l.html=



 =

 Instruções para entrar na lista, sair da lista e usar a lista em
 http://www.mat.puc-rio.br/~obmlistas/obm-l.htmlhttp://www.mat.puc-rio.br/%7Eobmlistas/obm-l.html
 =



Re: [obm-l] Re: [obm-l] Re: [obm-l] Fatoração de 5^1985 - 1.

2009-04-06 Por tôpico Carlos Nehab




Oi, Vidal (e Fabricio),

J que meu neto no est aqui em casa... :-)  e
como gostei tanto de suas continhas de cabea, fucei um site que tenho
certeza que vocs vo gostar Tem coisas surreais
http://www.leyland.vispa.com/numth/factorization/main.htm

Abraos,
Nehab
(
*Vidal escreveu:
Caro Fabrcio,
  
Eu tambm passei por esta etapa (produto de dois polinmios de grau 2)
durante o "pequeno" tempo que pensei na soluo, depois de "provocado"
pelo Nehab. Mas infelizmente os fatores no eram inteiros.
  
Abraos,
Vidal.
  
:: vi...@mail.com
  
  
  
  2009/4/6 fabrici...@usp.br fabrici...@usp.br
  Vidal,
muito boa a sacada.

Eu tinha tentado escrever como o produto de dois polinmios de grau 2,
sem sucesso.
Parabns pela soluo.

Um abrao.

.




On Apr 6, 2009, at 03:21 , *Vidal wrote:


Caros Fabrcio e Nehab,
  
Achar um fator foi fcil, o problema foi "quebrar" o quociente nos
outros dois.
  
Fiz assim:
  
5^1985 - 1 = (5^397)^5 - 1
  
Seja x = 5^397.
  
Ento queremos fatorar x^5 - 1 que, de imediato, resulta em (x - 1)
(x^4 + x^3 + x^2 + x + 1), ou seja, um dos fatores  5^397 - 1.
  
Falta fatorar x^4 + x^3 + x^2 + x + 1 de uma forma conveniente.
  
Aps um tempinho (pouca coisa, at no Fla x Flu no Maracan estava
rabiscando...), tive a idia de tentar escrever a expresso como uma
adequada diferena de dois quadrados. Caso conseguisse, o problema
estaria resolvido, pois um fator seria a soma e outro, a diferena.
  
Arbitrei o primeiro quadrado como (x^2 + ax + 1)^2, que j geraria o
termo de quarto grau e o termo independente corretos.
  
E coloquei o segundo quadrado como 5x(x+b)^2, pois como x = 5^397, 5x =
5^398 seria um quadrado perfeito.
  
Igualando as expresses (e rezando para encontrar valores de a e b
compatveis), veio:
  
(x^2 + ax + 1)^2 - 5x(x+b)^2 = x^4 + (2a -5)x^3 + (a^2 - 10b + 2)x^2 +
(2a - 5b^2)x + 1 = x^4 + x^3 + x^2 + x + 1
  
Assim:
  
2a -5 = 1 = a = 3
a^2 - 10b + 2 = 1 = b = 1
  
Agora era hora da ona beber gua:
  
2a - 5b^2 = 1
  
Mas a = 3 e b = 1 satisfazem ! Eureka !
  
x^4 + x^3 + x^2 + x + 1 = (x^2 + 3x + 1)^2 - 5x(x+1)^2
  
Substituindo x por 5^397:
  
((5^397)^2 + 3*5^397 +1)^2 - 5*5^397*(5^397 + 1)^2 =
  
= ((5^397)^2 + 3*5^397 +1)^2 - 5^398*(5^397 + 1)^2 (diferena de
quadrados) =
  
= (((5^397)^2 + 3*5^397 +1) - 5^199*(5^397 + 1)) * (((5^397)^2 +
3*5^397 +1) + 5^199*(5^397 + 1)) (produto da diferena pela soma) =
  
= (5^794 - 5^596 + 3*5^397 - 5^199 + 1) * (5^794 + 5^596 + 3*5^397 +
5^199 + 1)
  
Os trs fatores so claramente maiores que 5^100, conforme solicitado.
  
Ento:
  
5^1985 -1 = (5^397 - 1) * (5^794 - 5^596 + 3*5^397 - 5^199 + 1) *
(5^794 + 5^596 + 3*5^397 + 5^199 + 1)
  
Como j so trs da manh e j perdi o sono mesmo, resolvi fazer umas
"continhas de cabea", tal como o Ralph fez outro dia desses...
  
5^397-1 = 2 x 2 x 1.043.801.929 x 7.768.438.039 x C258
  
5^794 -5^596 + 3*5^397 - 5^199 + 1 = 71 x 399.091.951.801 x C542
  
5^794 + 5^596 + 3*5^397 + 5^199 + 1 = 11 x 146.891 x C549
  
Logo:
  
5^1985 -1 = 2 x 2 x 11 x 71 x 146.891 x 1.043.801.929 x 7.768.438.039 x
399.091.951.801 x C258 x C542 x C549
  
(onde Cn so nmeros compostos de n algarismos).
  
A fatorao de C258, C542 e C549 fica como exerccio ...
  
:)
  
Abraos,
Vidal.
  
P.S. Nehab: Apesar de no nos conhecermos pessoalmente, temos um grande
amigo em comum: o Manuel Martins Filho, professor de Informtica da
Carioca ! Abraos !
  
:: vi...@mail.com
  
***
  
2009/4/5 Carlos Nehab ne...@infolink.com.br
Oi, gente,
  
Fabricio postou este interessante problema e aparentemente ningum deu
muita bola, talvez achando que  bvio.
No achei bvio no. Quem resolveu?
  
Abraos,
Nehab
  
  fabrici...@usp.br escreveu:
  
Caros colegas,

mexendo em algumas listas antigas de exerccios, um me chamou muito a
ateno.
Pede pra fatorar 5^1985 - 1 num produto de trs inteiros maiores que
5^100.

Pra facilitar um possvel avano, 1985 pode ser escrito como 5 x 397
(ambos primos).

.
=
Instrues para entrar na lista, sair da lista e usar a lista em
http://www.mat.puc-rio.br/~obmlistas/obm-l.html
=

  
=
Instrues para entrar na lista, sair da lista e usar a lista em http://www.mat.puc-rio.br/~obmlistas/obm-l.html
=
  





=


Instrues para entrar na lista, sair da lista e usar a lista em
http://www.mat.puc-rio.br/~obmlistas/obm-l.html
=

Re: [obm-l] Fatoração de 5^1985 - 1 .

2009-04-05 Por tôpico Carlos Nehab




Oi, gente,

Fabricio postou este interessante problema e aparentemente ningum deu
muita bola, talvez achando que  bvio. 
No achei bvio no. Quem resolveu? 

Abraos, 
Nehab

fabrici...@usp.br escreveu:
Caros colegas,
  
  
mexendo em algumas listas antigas de exerccios, um me chamou muito a
ateno.
  
Pede pra fatorar 5^1985 - 1 num produto de trs inteiros maiores que
5^100.
  
  
Pra facilitar um possvel avano, 1985 pode ser escrito como 5 x 397
(ambos primos).
  
  
.
  
=
  
Instrues para entrar na lista, sair da lista e usar a lista em
  
http://www.mat.puc-rio.br/~obmlistas/obm-l.html
  
=
  
  



=
Instruções para entrar na lista, sair da lista e usar a lista em
http://www.mat.puc-rio.br/~obmlistas/obm-l.html
=


[obm-l] Fatoração de 5^1985 - 1.

2009-04-01 Por tôpico fabrici...@usp.br

Caros colegas,

mexendo em algumas listas antigas de exercícios, um me chamou muito a  
atenção.
Pede pra fatorar 5^1985 - 1 num produto de três inteiros maiores que  
5^100.


Pra facilitar um possível avanço, 1985 pode ser escrito como 5 x 397  
(ambos primos).


.
=
Instruções para entrar na lista, sair da lista e usar a lista em
http://www.mat.puc-rio.br/~obmlistas/obm-l.html
=


Re: [obm-l] Re: [obm-l] Fatoração

2007-01-24 Por tôpico Bruna Carvalho

Oi Marcelo então na minha apostilas está escrito exatamente assim
fatore x+1, para x=0.
la tem uma reposta bem feia feia, cheia de radicais.

=
Instruções para entrar na lista, sair da lista e usar a lista em
http://www.mat.puc-rio.br/~nicolau/olimp/obm-l.html
=


[obm-l] Re: [obm-l] Re: [obm-l] Fatoração

2007-01-24 Por tôpico Carlos Gomes
Bruna eh estranha a sua pergunta, mas talvez o autor deseja  que você faça o 
seguinte:


x+1=[raizcúbica(x)]^3 + [raizcúbica(1)]^3 .

agora use a identidade  a^3+b^3=(a+b).(a^2-ab+b^2) , fazendo 
a=raizcúbica(x)] e b=raizcúbica(1)]


daí você obtém x+1=[raizcúbica(x)]^3 + [raizcúbica(1)]^3 = [raizcúbica(x) + 
1].[(raizcúbica(x))^2-raizcúbica(x)+1].



Valew  Cgomes



- Original Message - 
From: Bruna Carvalho [EMAIL PROTECTED]

To: obm-l@mat.puc-rio.br
Sent: Wednesday, January 24, 2007 6:55 PM
Subject: Re: [obm-l] Re: [obm-l] Fatoração


Oi Marcelo então na minha apostilas está escrito exatamente assim
fatore x+1, para x=0.
la tem uma reposta bem feia feia, cheia de radicais.

=
Instruções para entrar na lista, sair da lista e usar a lista em
http://www.mat.puc-rio.br/~nicolau/olimp/obm-l.html
=


--
No virus found in this incoming message.
Checked by AVG Free Edition.
Version: 7.1.410 / Virus Database: 268.17.8/649 - Release Date: 23/1/2007


=
Instruções para entrar na lista, sair da lista e usar a lista em
http://www.mat.puc-rio.br/~nicolau/olimp/obm-l.html
=


[obm-l] Re: [obm-l] Fatoração

2007-01-23 Por tôpico Marcelo Salhab Brogliato
Olá Bruna,
não entendi direito o q quer dizer com fatorar x+1... acredito que seja 
escrever de uma forma mais complexa..

por exemplo:

(x^2 - 1)/(x - 1) ... claro que esta bem facil concluir que é x+1, mas já é uma 
fatoração né?

1 = sen(25) / cos(65) = sen(50) / [2 * cos(25) * cos(65)]

x = sen(arcsen(x)) = sen[2arcsen(x)] / { 2 * cos[arcsen(x)] }

x + 1 = sen[2arcsen(x)] / { 2 * cos[arcsen(x)] } + sen(50) / [2 * cos(25) * 
cos(65)] = { 2 * cos(25) * cos(65) * sen[2arcsen(x)] + 2 * sen(50) * 
sen[2arcsen(x)] } / { 2 * cos[arcsen(x)] * 2 * cos(25) * cos(65) }

x + 1 = { cos(25) * cos(65) * sen[2arcsen(x)] + sen(50) * sen[2arcsen(x)] } / { 
2 * cos[arcsen(x)] * cos(25) * cos(65) }

bom.. nao sei c era isso o esperado.. mas.. :)

abraços,
Salhab

  - Original Message - 
  From: Bruna Carvalho 
  To: obm-l@mat.puc-rio.br 
  Sent: Monday, January 22, 2007 2:18 PM
  Subject: [obm-l] Fatoração


  Fatorar x+1, para x=0.

  -- 
  Bjos, 
  Bruna 

[obm-l] Fatoração

2007-01-22 Por tôpico Bruna Carvalho

Fatorar x+1, para x=0.

--
Bjos,
Bruna


Re: [obm-l] Fatoração

2006-10-16 Por tôpico Carlos Victor


Olá  Bruna ,
Adicione  e  subtraia  os  fatores : x^10 ,x^9 , ... x  .Depois  é só 
agrupar  os  fatores : x^11+x^10+x^9 , -(x^10+x^9+x^8)  e  assim 
por  diante  ; onde   o fator  x^2+x+1  será  comum . Conclusão



x^11 + x^7 + 1 = ( x^2+x+1)(x^9-x^8+x^6-x^4+x^3-x+1) , ok ?

[]´s  Carlos  Victor



At 14:05 14/10/2006, Bruna Carvalho wrote:

Fatorar:

P(x) = x^11 + x^7 + 1.




=
Instruções para entrar na lista, sair da lista e usar a lista em
http://www.mat.puc-rio.br/~nicolau/olimp/obm-l.html
=


[obm-l] Fatoração

2006-10-14 Por tôpico Bruna Carvalho
Fatorar: P(x) = x^11 + x^7 + 1.


Re: [obm-l] Re:[obm-l] Fatoração

2006-02-22 Por tôpico Marcio M Rocha

Salhab [ k4ss ] escreveu:



 
(a+b+c)^4 = 1

*fatorando*.. temos:
a^4 + b^4 + c^4 + 4 [(ab)^2 + (bc)^2 + (ac)^2 + 2abc] = 1
a^4 + b^4 + c^4 + 4 * 1/4 = 1
a^4 + b^4 + c^4 = 0
 

 
 

Sem querer ser chato, gostaria de fazer uma pequeníssima correção nas 
palavras: no caso, o correto é *expandindo*, e não fatorando.


Márcio.


=
Instruções para entrar na lista, sair da lista e usar a lista em
http://www.mat.puc-rio.br/~nicolau/olimp/obm-l.html
=


[obm-l] Fatoração

2006-02-21 Por tôpico Dymitri Cardoso Leão
Oh só galera, me pareceu fácil, mas não estou enxergando alguma coisa, e 
empaquei nesta questão.


Se a+b+c=1 e a^2 + b^2 + c^2 =0, calcule a^4+b^4+c^4. Sei que a resposta é 
1/2. Depois de muita manipulação algébrica, cheguei em uma expressão 
envolvendo a soma pedida e o produto abc, deu -1/2 + 4abc, mas não consegui 
tirar este produto dos dados. Ajudem aí.


Notação: x^y (x elevado a y)

_
Com o MSN Spaces você divide seu blog, suas fotos, sua lista de música e 
muito mais com seus amigos! Crie já o seu espaço online e com seus amigos! E 
só entra no   http://spaces.msn.com/


=
Instruções para entrar na lista, sair da lista e usar a lista em
http://www.mat.puc-rio.br/~nicolau/olimp/obm-l.html
=


[obm-l] RE: [obm-l] Fatoração

2006-02-21 Por tôpico Rafael Bonifácio

Ops desculpe, mandei mensagens erradas...

_
MSN Busca: fácil, rápido, direto ao ponto.  http://search.msn.com.br

=
Instruções para entrar na lista, sair da lista e usar a lista em
http://www.mat.puc-rio.br/~nicolau/olimp/obm-l.html
=


[obm-l] RES: [obm-l] Fatoração

2006-02-21 Por tôpico Artur Costa Steiner
Tem alguma coisa errada.  Se a,b, c sao reais, entao a^2 + b^2 + c^2 =0 se,
e somente se, a=b=c=0.
Artur

-Mensagem original-
De: [EMAIL PROTECTED] [mailto:[EMAIL PROTECTED]
nome de Dymitri Cardoso Leão
Enviada em: terça-feira, 21 de fevereiro de 2006 16:34
Para: obm-l@mat.puc-rio.br
Assunto: [obm-l] Fatoração


Oh só galera, me pareceu fácil, mas não estou enxergando alguma coisa, e 
empaquei nesta questão.

Se a+b+c=1 e a^2 + b^2 + c^2 =0, calcule a^4+b^4+c^4. Sei que a resposta é 
1/2. Depois de muita manipulação algébrica, cheguei em uma expressão 
envolvendo a soma pedida e o produto abc, deu -1/2 + 4abc, mas não consegui 
tirar este produto dos dados. Ajudem aí.

Notação: x^y (x elevado a y)

_
Com o MSN Spaces você divide seu blog, suas fotos, sua lista de música e 
muito mais com seus amigos! Crie já o seu espaço online e com seus amigos! E

só entra no   http://spaces.msn.com/

=
Instruções para entrar na lista, sair da lista e usar a lista em
http://www.mat.puc-rio.br/~nicolau/olimp/obm-l.html
=

=
Instruções para entrar na lista, sair da lista e usar a lista em
http://www.mat.puc-rio.br/~nicolau/olimp/obm-l.html
=


[obm-l] RE: [obm-l] Fatoração

2006-02-21 Por tôpico Rafael Bonifácio

(a+b+c)^2= a^2+ab+ac+b^2+ba+bc+c^2+ca+cb= a^2+b^2+c^2+2ab+2ac+2cb


From: Dymitri Cardoso Leão [EMAIL PROTECTED]
Reply-To: obm-l@mat.puc-rio.br
To: obm-l@mat.puc-rio.br
Subject: [obm-l] Fatoração
Date: Tue, 21 Feb 2006 19:34:29 +

Oh só galera, me pareceu fácil, mas não estou enxergando alguma coisa, e 
empaquei nesta questão.


Se a+b+c=1 e a^2 + b^2 + c^2 =0, calcule a^4+b^4+c^4. Sei que a resposta é 
1/2. Depois de muita manipulação algébrica, cheguei em uma expressão 
envolvendo a soma pedida e o produto abc, deu -1/2 + 4abc, mas não consegui 
tirar este produto dos dados. Ajudem aí.


Notação: x^y (x elevado a y)

_
Com o MSN Spaces você divide seu blog, suas fotos, sua lista de música e 
muito mais com seus amigos! Crie já o seu espaço online e com seus amigos! E 
só entra no   http://spaces.msn.com/


=
Instruções para entrar na lista, sair da lista e usar a lista em
http://www.mat.puc-rio.br/~nicolau/olimp/obm-l.html
=

_
MSN Busca: fácil, rápido, direto ao ponto.  http://search.msn.com.br

=
Instruções para entrar na lista, sair da lista e usar a lista em
http://www.mat.puc-rio.br/~nicolau/olimp/obm-l.html
=


[obm-l] Re:[obm-l] Fatoração

2006-02-21 Por tôpico Salhab \[ k4ss \]

Olá,

a+b+c = 1
(a+b+c)^2 = 1
a^2 + b^2 + c^2 + 2(ab + ac + bc) = 1
mas a^2 + b^2 + c^2 = 0, logo:

ab + ac + bc = 1/2

(ab+ac+bc)^2 = 1/4
(ab)^2 + (ac)^2 + (bc)^2 + 2(bca^2 + acb^2 + abc^2) = 1/4
(ab)^2 + (ac)^2 + (bc)^2 + 2abc(a+b+c) = 1/4
(ab)^2 + (ac)^2 + (bc)^2 + 2abc = 1/4

Ok!

(a+b+c)^4 = 1
fatorando.. temos:
a^4 + b^4 + c^4 + 4 [(ab)^2 + (bc)^2 + (ac)^2 + 2abc] = 1
a^4 + b^4 + c^4 + 4 * 1/4 = 1
a^4 + b^4 + c^4 = 0

po, absurdo.. rs rs! axo q errei alguma continha.. mas eh por ai!
soh conferir as coisas.. espero ter ajudado!
abraços,
Salhab


 Oh só galera, me pareceu fácil, mas não estou enxergando alguma coisa, e 
 empaquei nesta questão. 
 
 Se a+b+c=1 e a^2 + b^2 + c^2 =0, calcule a^4+b^4+c^4. Sei que a resposta é 
 1/2. Depois de muita manipulação algébrica, cheguei em uma expressão 
 envolvendo a soma pedida e o produto abc, deu -1/2 + 4abc, mas não consegui 
 tirar este produto dos dados. Ajudem aí. 
 
 Notação: x^y (x elevado a y) 
 
 _ 
 Com o MSN Spaces você divide seu blog, suas fotos, sua lista de música e 
 muito mais com seus amigos! Crie já o seu espaço online e com seus amigos! E 
 só entra no http://spaces.msn.com/ 
 
 = 
 Instruções para entrar na lista, sair da lista e usar a lista em 
 http://www.mat.puc-rio.br/~nicolau/olimp/obm-l.html 
 = 
 


[obm-l] Re:[obm-l] RES: [obm-l] Fatoração

2006-02-21 Por tôpico Salhab \[ k4ss \]

caraca.. nem me liguei nisso! rs rs.. eh verdade
por isso cheguei que a^4 + b^4 + c^4 = 0...
hmm.. axo q deve ser ao contrario:

a+b+c = 0
a^2 + b^2 + c^2 = 1

dai sim teriamos um resultado coerente!
abraços
Salhab

 Tem alguma coisa errada. Se a,b, c sao reais, entao a^2 + b^2 + c^2 =0 se, 
 e somente se, a=b=c=0. 
 Artur 
 
 -Mensagem original- 
 De: [EMAIL PROTECTED] [mailto:[EMAIL PROTECTED] 
 nome de Dymitri Cardoso Leão 
 Enviada em: terça-feira, 21 de fevereiro de 2006 16:34 
 Para: obm-l@mat.puc-rio.br 
 Assunto: [obm-l] Fatoração 
 
 
 Oh só galera, me pareceu fácil, mas não estou enxergando alguma coisa, e 
 empaquei nesta questão. 
 
 Se a+b+c=1 e a^2 + b^2 + c^2 =0, calcule a^4+b^4+c^4. Sei que a resposta é 
 1/2. Depois de muita manipulação algébrica, cheguei em uma expressão 
 envolvendo a soma pedida e o produto abc, deu -1/2 + 4abc, mas não consegui 
 tirar este produto dos dados. Ajudem aí. 
 
 Notação: x^y (x elevado a y) 
 
 _ 
 Com o MSN Spaces você divide seu blog, suas fotos, sua lista de música e 
 muito mais com seus amigos! Crie já o seu espaço online e com seus amigos! E 
 
 só entra no http://spaces.msn.com/ 
 
 = 
 Instruções para entrar na lista, sair da lista e usar a lista em 
 http://www.mat.puc-rio.br/~nicolau/olimp/obm-l.html 
 = 
 
 = 
 Instruções para entrar na lista, sair da lista e usar a lista em 
 http://www.mat.puc-rio.br/~nicolau/olimp/obm-l.html 
 = 
 


Re: [obm-l] Fatoração

2006-02-21 Por tôpico fabio
 Oh só galera, me pareceu fácil, mas não estou enxergando alguma coisa, e
 empaquei nesta questão.

 Se a+b+c=1 e a^2 + b^2 + c^2 =0, calcule a^4+b^4+c^4. Sei que a resposta é
 1/2. Depois de muita manipulação algébrica, cheguei em uma expressão
 envolvendo a soma pedida e o produto abc, deu -1/2 + 4abc, mas não
 consegui
 tirar este produto dos dados. Ajudem aí.
 [...]

A questão está errada mesmo (e a conta que você fez está certa). Faltam
informações.

[]s,

-- 
Fábio Dias Moreira
=
Instruções para entrar na lista, sair da lista e usar a lista em
http://www.mat.puc-rio.br/~nicolau/olimp/obm-l.html
=


Re: [obm-l] fatoração...

2006-02-13 Por tôpico Eduardo Wilner
  Prezado Carlos GomesAcho  que o problema fica mais "leve" se levarmos em conta que tanto os  termos do primeiro fator, A1+A2+A3, quanto os do segundo  B1+B2+B3, podem ser obtidos de um deles pela permutaçao ciclica entre  a, b e c , respectivamente.Eh imediato que Ai.Bi=1 para i=1,2,3 , logo Soma(com i de 1 a 3) de (AiBi) =3.  O problema reside nos produtos cruzados.Note que, p.ex., A1+A2 =(a-b)/c + (b-c)/a = 2b(c-a)/(ac) que multiplicado pelo "seu cruzado", B3, resulta em  (A1+A2).B3 = 2b^2/(ac) = -2.(b/a + b/c) ( lembrando que b = -(a+c) ).  Para os demais produtos cruzados basta fazer a permutação ciclica e obteremos para   !
p; 
Soma(i , j ,k diferentes entre si, de 1 a 3) de (Ai+Aj).Bk =  = -2.(b/a + b/c + c/b + c/a + a/c + a/b) =  -2.[(b+c)/a + (b+a)/c + (c+a)/b] = 6Abraços  Wilner  Carlos Gomes [EMAIL PROTECTED] escreveu:  V se alguem me ajuda com essa...Se a+b+c=0, qual o valor da expressão [(a-b)/c +   (b-c)/a + (c-a)/b].[c/(a-b) + a/(b-c) + b/(c-a)]
o gabarito dá como resposta 9...tá dando muito   trabalho...v se alguem descobre algum atalho...valew...um abraço à   todosCgomes--  Esta mensagem foi verificada pelo sistema de anti-virus eacredita-se estar livre de perigo.  
		 
Yahoo! Acesso Grátis 
Internet rápida e grátis. Instale o discador agora!

[obm-l] fatoração...

2006-02-10 Por tôpico Carlos Gomes



V se alguem me ajuda com essa...

Se a+b+c=0, qual o valor da expressão [(a-b)/c + 
(b-c)/a + (c-a)/b].[c/(a-b) + a/(b-c) + b/(c-a)]

o gabarito dá como resposta 9...tá dando muito 
trabalho...v se alguem descobre algum atalho...valew...um abraço à 
todos

Cgomes--
Esta mensagem foi verificada pelo sistema de anti-virus e 
 acredita-se estar livre de perigo.



[obm-l] Re: [obm-l] fatoração...

2006-02-10 Por tôpico Marcelo Salhab Brogliato



Olá,
cara, se for questao de teste, faca 
assim:
a+b+c = 0... hmm.. a=3, b=-2, c=-1 ... 
ok!
(a-b)/c = (3 + 2)/(-1) = -5
(b-c)/a = (-2+1)/3 = -1/3
(c-a)/b = (-1-3)/(-2) = 2

-5 -1/3 + 2 = -3 -1/3 = -10/3

-1/5 - 3 + 1/2 = [ -2 - 30 + 5 ] / 10 = 
-27/10

(-10/3) * (-27/10) = 9

mas, se for questao dissertativa.. axo q eh braco 
mesmo! pelo menos nao vi uma saida simples...

abraços,
Salhab

  - Original Message - 
  From: 
  Carlos 
  Gomes 
  To: obm-l@mat.puc-rio.br 
  Sent: Friday, February 10, 2006 9:51 
  AM
  Subject: [obm-l] fatoração...
  
  V se alguem me ajuda com essa...
  
  Se a+b+c=0, qual o valor da expressão [(a-b)/c + 
  (b-c)/a + (c-a)/b].[c/(a-b) + a/(b-c) + b/(c-a)]
  
  o gabarito dá como resposta 9...tá dando muito 
  trabalho...v se alguem descobre algum atalho...valew...um abraço à 
  todos
  
  Cgomes-- Esta mensagem foi 
  verificada pelo sistema de anti-virus e acredita-se estar livre de perigo. 



[obm-l] Re:[obm-l] fatoração...

2006-02-10 Por tôpico Luiz H\. Barbosa

Esse tipo de problema sempre da um trabalhinho.Mas eu nãotentaria a resolução genérica em uma prova de multipla escolha,tascaria 3 números cujo a soma da ZERO e pronto!

Chamei a primeira parte de I e a segunda de II.
Observe que ,
c(b-c)(c-a) = c(bc-ab-c^2 + ac) = c(-ab+c(b-c+a)) = c(-ab-2c^2) = -abc -2c^3
c(b-c)(c-a) = -abc -2c^3 (i)
O mesmo raciocinio serve para concluir que :
a(a-b)(c-a) = -abc-2a^3 (ii)
b(a-b)(b-c) = -abc-2b^3 (iii)

Aparte II fica :
II = [(i)+(ii)+(iii)]/(a-b)(b-c)(c-a) = [-3abc -2(a^3 + b^3 +c^3)]/(a-b)(b-c)(c-a) 

Agora veja que :
(a-b)(b-c)(c-a) = abc - b*a^2 - a*c^2 + c*a^2 + c*b^2 + a*b^2 + b*c^2 -abc

A parte I fica:
I=[ab(a-b) + bc(b-c) + ac(c-a)]/abc
I=[b*a^2 -a*b^2 +c*b^2 -b*c^2 + a*c^2 - c*a^2]/abc
I=[b*a^2 -a*b^2 +c*b^2 -b*c^2 + a*c^2 - c*a^2 + abc - abc]/abc
I= -[(a-b)(b-c)(c-a)]abc

Agora multiplicando I*II :
I*II=[3abc +2(a^3 + b^3 +c^3)]/abc = 3+ 2[(a^3 + b^3 +c^3)]/abc

Se vc fizer (a+b+c)^3 = 0 e isolar de um lado a^3 + b^3 +c^3, vai encontrar :
a^3 + b^3 +c^3 = -3(a*b^2 + a*c^2 + b*a^2 + c*a^2 + b*c^2 + c*b^2 + 2abc)

[a^3 + b^3 +c^3]abc = -3(b/c + c/b + a/c + a/b + c/a + b/a + 2)
Observe agora que :
a/c = -1 -b/c 
c/b = -1-a/b
c/a = -1-b/a
Substiruindo :
[a^3 + b^3 +c^3]abc = -3(-3 + 2) = 3

Finalmente:
I*II = 3 + 2*3 = 9


-- Início da mensagem original --- 

De: [EMAIL PROTECTED] 
Para: obm-l@mat.puc-rio.br 
Cc: 
Data: Fri, 10 Feb 2006 09:51:27 -0200 
Assunto: [obm-l] fatoração... 

 V se alguem me ajuda com essa... 
 
 Se a+b+c=0, qual o valor da expressão [(a-b)/c + (b-c)/a + (c-a)/b].[c/(a-b) + a/(b-c) + b/(c-a)] 
 
 o gabarito dá como resposta 9...tá dando muito trabalho...v se alguem descobre algum atalho...valew...um abraço à todos 
 
 Cgomes 
 -- 
 Esta mensagem foi verificada pelo sistema de anti-virus e 
 acredita-se estar livre de perigo. 
 
 


[obm-l] Re: [obm-l] Re:[obm-l] fatoração...

2006-02-10 Por tôpico Carlos Gomes



valew Luiz muito obrigado!

  - Original Message - 
  From: 
  Luiz H. 
  Barbosa 
  To: obm-l 
  Sent: Friday, February 10, 2006 7:53 
  PM
  Subject: [obm-l] Re:[obm-l] 
  fatoração...
  
  
  Esse tipo de problema sempre da um trabalhinho.Mas eu nãotentaria a 
  resolução genérica em uma prova de multipla escolha,tascaria 3 números cujo a 
  soma da ZERO e pronto!
  
  Chamei a primeira parte de I e a segunda de II.
  Observe que ,
  c(b-c)(c-a) = c(bc-ab-c^2 + ac) = c(-ab+c(b-c+a)) = c(-ab-2c^2) = -abc 
  -2c^3
  c(b-c)(c-a) = -abc -2c^3 (i)
  O mesmo raciocinio serve para concluir que :
  a(a-b)(c-a) = -abc-2a^3 (ii)
  b(a-b)(b-c) = -abc-2b^3 (iii)
  
  Aparte II fica :
  II = [(i)+(ii)+(iii)]/(a-b)(b-c)(c-a) = [-3abc -2(a^3 + b^3 
  +c^3)]/(a-b)(b-c)(c-a) 
  
  Agora veja que :
  (a-b)(b-c)(c-a) = abc - b*a^2 - a*c^2 + c*a^2 + c*b^2 + a*b^2 + b*c^2 
  -abc
  
  A parte I fica:
  I=[ab(a-b) + bc(b-c) + ac(c-a)]/abc
  I=[b*a^2 -a*b^2 +c*b^2 -b*c^2 + a*c^2 - c*a^2]/abc
  I=[b*a^2 -a*b^2 +c*b^2 -b*c^2 + a*c^2 - c*a^2 + abc - abc]/abc
  I= -[(a-b)(b-c)(c-a)]abc
  
  Agora multiplicando I*II :
  I*II=[3abc +2(a^3 + b^3 +c^3)]/abc = 3+ 2[(a^3 + b^3 
  +c^3)]/abc
  
  Se vc fizer (a+b+c)^3 = 0 e isolar de um lado a^3 + b^3 +c^3, 
  vai encontrar :
  a^3 + b^3 +c^3 = -3(a*b^2 + a*c^2 + b*a^2 + c*a^2 + b*c^2 + c*b^2 + 
  2abc)
  
  [a^3 + b^3 +c^3]abc = -3(b/c + c/b + a/c + a/b + c/a + b/a + 
  2)
  Observe agora que :
  a/c = -1 -b/c 
  c/b = -1-a/b
  c/a = -1-b/a
  Substiruindo :
  [a^3 + b^3 +c^3]abc = -3(-3 + 2) = 3
  
  Finalmente:
  I*II = 3 + 2*3 = 9
  
  
  -- Início da mensagem original --- 
  
  De: [EMAIL PROTECTED] 
  Para: obm-l@mat.puc-rio.br 
  Cc: 
  Data: Fri, 10 Feb 2006 09:51:27 -0200 
  Assunto: [obm-l] fatoração... 
  
   V se alguem me ajuda com essa... 
   
   Se a+b+c=0, qual o valor da expressão [(a-b)/c + (b-c)/a + 
  (c-a)/b].[c/(a-b) + a/(b-c) + b/(c-a)] 
   
   o gabarito dá como resposta 9...tá dando muito trabalho...v se 
  alguem descobre algum atalho...valew...um abraço à todos 
   
   Cgomes 
   -- 
   Esta mensagem foi verificada pelo sistema de anti-virus e 
   acredita-se estar livre de perigo. 
   
   -- Esta mensagem foi verificada pelo sistema de 
  anti-virus e acredita-se estar livre de perigo. --
Esta mensagem foi verificada pelo sistema de anti-virus e 
 acredita-se estar livre de perigo.



Re: [obm-l] fatoração...

2006-02-10 Por tôpico Danilo Nascimento
Ola Carlos,  observe que a expressao eh da forma (x+y+z)(1/x + 1/y + 1/z).  Sabemos que para x0 y0 e z0 a expressao acima assume valor =9  (eh facil de demonstrar) agora fazendo (a-b)/c0, (b-c)/a0 e (c-a)0  e somando as expressoes vai chegar que a+b+c0 o que contraria a sua hipotese de a+b+c=0 logo so vale a igualdade. E=9Carlos Gomes [EMAIL PROTECTED] escreveu:  V se alguem me ajuda com essa...Se a+b+c=0, qual o valor da expressão [(a-b)/c + (b-c)/a + (c-a)/b].[c/(a-b) + a/(b-c) + b/(c-a)]o gabarito dá como resposta 9...tá dando muito trabalho...v se alguem descobre algum atalho...valew...um abraço à todosCgomes-- Esta mensagem foi verificada pelo sistema de anti-virus e acredita-se estar livre de perigo. 
		 
Yahoo! Search 
Dê uma espiadinha e saiba tudo sobre o Big Brother Brasil.

[obm-l] Fatoração?

2005-10-27 Por tôpico Raul Ribeiro

 Boa tarde!

 Essa é da Opm-02 (Alguém sabe onde encontrar os gabaritos das opm's?)

 Prove que a equação abaixo tem infinitas soluções inteiras positivas?

 x^3 + 2y^3 + 4z^3 - 6xyz = 1 


=
Instruções para entrar na lista, sair da lista e usar a lista em
http://www.mat.puc-rio.br/~nicolau/olimp/obm-l.html
=


Re: [obm-l] Fatoração?

2005-10-27 Por tôpico Claudio Buffara
Title: Re: [obm-l] Fatoração?



Por que?

on 27.10.05 18:38, Iuri at [EMAIL PROTECTED] wrote:

a^3 + b^3 + c^3 - 3abc = (a+b+c)(a^2 + b^2 + c^2 -ab -ac -bc)
Usando essa identidade, ta provado.


Em 27/10/05, Raul Ribeiro [EMAIL PROTECTED] escreveu:
 Boa tarde!

 Essa é da Opm-02 (Alguém sabe onde encontrar os gabaritos das opm's?)

 Prove que a equação abaixo tem infinitas soluções inteiras positivas?

 x^3 + 2y^3 + 4z^3 - 6xyz = 1

= 
Instruções para entrar na lista, sair da lista e usar a lista em
http://www.mat.puc-rio.br/~nicolau/olimp/obm-l.html
= 








Re: [obm-l] Fatoração?

2005-10-27 Por tôpico Iuri
Ops, desculpa, pensei numa outra coisa. Vo ver se eu faço aqui.Em 27/10/05, Claudio Buffara [EMAIL PROTECTED]
 escreveu:




Por que?

on 27.10.05 18:38, Iuri at [EMAIL PROTECTED] wrote:

a^3 + b^3 + c^3 - 3abc = (a+b+c)(a^2 + b^2 + c^2 -ab -ac -bc)
Usando essa identidade, ta provado.


Em 27/10/05, Raul Ribeiro [EMAIL PROTECTED] escreveu:
 Boa tarde!

 Essa é da Opm-02 (Alguém sabe onde encontrar os gabaritos das opm's?)

 Prove que a equação abaixo tem infinitas soluções inteiras positivas?

 x^3 + 2y^3 + 4z^3 - 6xyz = 1

= 
Instruções para entrar na lista, sair da lista e usar a lista em
http://www.mat.puc-rio.br/~nicolau/olimp/obm-l.html
= 










[obm-l] Re:[obm-l] Fatoração na questã o do DIVISOR

2005-06-26 Por tôpico vinicius
 Não entendi como o Cláudio fatorou o polonômio a^33-a^19-a^17-1
 abaixo. Tem alguma regra geral para essa fatoração?
 
  Aklias, sera que da para fatorar o polinomio
  a^33-a^19-a^17-1 ?
 
 Certamente. 
 Isso eh igual a (a + 1)*f(a), onde f(a) é mônico de grau 32.
 Aliás, isso dá uma solução mais natural para o problema original,
 com a = 2, pois mostra que além de ser ímpar, a expressão é
 divisível por 3.
 
 []s,
 Claudio.
 

Desculpe, pela simplicidade da pergunta, mas oq vem a ser monico de grau 32?
__
UOL Fone: Fale com o Brasil e o Mundo com at 90% de economia 
http://www.uol.com.br/fone




[obm-l] Fatoração na questão do DIVISOR

2005-06-25 Por tôpico Igor O.A.
Não entendi como o Cláudio fatorou o polonômio a^33-a^19-a^17-1
abaixo. Tem alguma regra geral para essa fatoração?

  Aklias, sera que da para fatorar o polinomio
  a^33-a^19-a^17-1 ?
 
 Certamente. 
 Isso eh igual a (a + 1)*f(a), onde f(a) é mônico de grau 32.
 Aliás, isso dá uma solução mais natural para o problema original,
com a = 2,pois mostra que além de ser ímpar, a expressão é
divisível por 3.
 
[]s,
Claudio.
 

-- 
I G O R

Jesus ama você.

=
Instruções para entrar na lista, sair da lista e usar a lista em
http://www.mat.puc-rio.br/~nicolau/olimp/obm-l.html
=


[obm-l] RE: [obm-l] Fatoração na questão do DIVISOR

2005-06-25 Por tôpico kleinad2
Bem, respondendo especificamente à sua pergunta: se x for raiz de p(a),
então (a - x) divide p(a), e foi o que o Cláudio usou com x = -1.

De uma forma mais geral, se x for raiz de p(a) e q(a) for o polinômio 
irredutível
de x sobre o corpo base F (p e q são polinômios em F[a]), então q(a) divide
p(a) em F. Assim por exemplo qualquer polinômio p(a) com coeficientes racionais
tal que sqrt(2) seja raiz de p, então q(a) = a^2 - 2 (irredutível de sqrt(2)
sobre os racionais) é divisor de p(a) sobre os racionais.

[]s,
Daniel

 ''Não entendi como o Cláudio fatorou o polonômio a^33-a^19-a^17-1
 ''abaixo. Tem alguma regra geral para essa fatoração?
 ''
 ''  Aklias, sera que da para fatorar o polinomio
 ''  a^33-a^19-a^17-1 ?
 '' 
 '' Certamente. 
 '' Isso eh igual a (a + 1)*f(a), onde f(a) é mônico de grau 32.



=
Instruções para entrar na lista, sair da lista e usar a lista em
http://www.mat.puc-rio.br/~nicolau/olimp/obm-l.html
=


Re: [obm-l] Fatoração na questão do DIVISOR

2005-06-25 Por tôpico Marcos Martinelli
Você deve perceber que a=-1 é uma raiz desse polinômio de grau 33. E
usar Briot-Ruffini para concluir que o outro polinômio é mônico de
grau 32. Na verdade fatorar um polinômio é encontrar suas raízes.
Abraços!

Em 25/06/05, Igor O.A.[EMAIL PROTECTED] escreveu:
 Não entendi como o Cláudio fatorou o polonômio a^33-a^19-a^17-1
 abaixo. Tem alguma regra geral para essa fatoração?
 
  Aklias, sera que da para fatorar o polinomio
  a^33-a^19-a^17-1 ?
 
 Certamente.
 Isso eh igual a (a + 1)*f(a), onde f(a) é mônico de grau 32.
 Aliás, isso dá uma solução mais natural para o problema original,
 com a = 2,pois mostra que além de ser ímpar, a expressão é
 divisível por 3.
 
 []s,
 Claudio.
 
 --
 I G O R
 
 Jesus ama você.
 
 =
 Instruções para entrar na lista, sair da lista e usar a lista em
 http://www.mat.puc-rio.br/~nicolau/olimp/obm-l.html
 =


=
Instruções para entrar na lista, sair da lista e usar a lista em
http://www.mat.puc-rio.br/~nicolau/olimp/obm-l.html
=


[obm-l] Fatoração

2005-03-20 Por tôpico Daniela Yoshikawa
Olá!

Fatorar:

1) (a-b)c^3 - (a-c)b^3 + (b-c)a^3
2) a^4 - 2a^3b - 8a^2b^2 - 6ab^3 - b^4
3) a^3(a^2 - 7)^2 - 36a
4) a+b+c = 0 - (a^5 + b^5 + c^5)/5 = (a^3 + b^3 + c^3)/3 . (a^2 + b^2 + c^2)/2
5) Prove that if a/(b-c) + b/(c-a) + c/(a-b) = 0, where ab, ac, bc, thena/(b-c)^2 + b/(c-a)^2 + c/(a-b)^2 = 0

Obrigada,Daniele.
		Yahoo! Mail - Com 250MB de espaço.  Abra sua conta!

[obm-l] fatoração

2005-03-13 Por tôpico marcio aparecido
alguem me ajuda com essa fatoração que segue:

(x^ -8) - (y^ -8) / (x^ -2 * y^ -2) * (x^ -4 + y^ -4)

=
Instruções para entrar na lista, sair da lista e usar a lista em
http://www.mat.puc-rio.br/~nicolau/olimp/obm-l.html
=


Re: [obm-l] fatoração

2005-03-13 Por tôpico fabiodjalma

 O primeiro vezes não deveria ser mais? 

Em (14:28:24), obm-l@mat.puc-rio.br escreveu: 


alguem me ajuda com essa fatoração que segue: 
 
(x^ -8) - (y^ -8) / (x^ -2 * y^ -2) * (x^ -4 + y^ -4) 
 
= 
Instruções para entrar na lista, sair da lista e usar a lista em 
http://www.mat.puc-rio.br/~nicolau/olimp/obm-l.html 
= 
 
-- 


Re: [obm-l] fatoração

2005-03-13 Por tôpico marcio aparecido
no meu livro tá vezes mesmo

=
Instruções para entrar na lista, sair da lista e usar a lista em
http://www.mat.puc-rio.br/~nicolau/olimp/obm-l.html
=


Re: [obm-l] fatoração

2005-03-13 Por tôpico fabiodjalma
 Então transforme (1/x)^8 - (1/y)^8 em 
[(1/x)^4 - (1/y)^4].[(1/x)^4 + (1/y)^4] 


Em (15:38:53), obm-l@mat.puc-rio.br escreveu: 


no meu livro tá vezes mesmo 
 
= 
Instruções para entrar na lista, sair da lista e usar a lista em 
http://www.mat.puc-rio.br/~nicolau/olimp/obm-l.html 
= 
 
-- 


[obm-l] Fatoração

2004-07-10 Por tôpico Maurizio
Quem puder me ajudar a fatorar isso aqui agradeço antecipadamente:
=(x-1)(x-2)(x-3)(x-4)-15
=(xy-1)(x-1)(y+1)-xy
=a^12+b^12
Fatorar em função de A, depois em função de B:
=a^2+2ab+b^2-x^2-6x-9

MauZ
=
Instruções para entrar na lista, sair da lista e usar a lista em
http://www.mat.puc-rio.br/~nicolau/olimp/obm-l.html
=


[obm-l] Fatoração II

2004-05-10 Por tôpico niski
Vamos ver agora.
Fatore x^6 + x^3 + 1
Obs. Para evitar respostas do tipo
1*(x^6 + x^3 + 1) ou sobre o que realmente significa fatorar,
eu cheguei numa expressao do tipo
(f(x) - Ax + B)(f(x) - Cx + B)(f(x) - Dx + C) onde A,B,C,D sao 
constantes e f é uma funcao...

Depois eu coloco exatamente qual foi a minha resposta. Mas sera que 
alguem sabe um jeito simples de fazer?

Abraços

--
Niski - http://www.linux.ime.usp.br/~niski
[upon losing the use of his right eye]
Now I will have less distraction
Leonhard Euler
=
Instruções para entrar na lista, sair da lista e usar a lista em
http://www.mat.puc-rio.br/~nicolau/olimp/obm-l.html
=


[obm-l] Re: [obm-l] Fatoração II

2004-05-10 Por tôpico Cláudio \(Prática\)
p(x) = x^6 + x^3 + 1 = (x^9 - 1)/(x^3 - 1)
Ou seja, as raízes de p(x) são as raízes nonas da unidade com exceção de 1,
exp(i*2pi/3) e exp(i*4pi/3).

Seja w = exp(i*2pi/9).
Então as raízes de x^6 + x^3 + 1 são:
w, w^2, w^4, w^(-1), w^(-2) e w^(-4).

w + w^(-1) = 2*cos(2pi/9) = A
w^2 + w^(-2) = 2*cos(4pi/9) = B
w^4 + w^(-4) = 2*cos(8pi/9) = C

Logo:
p(x) = (x - w)*(x - w^(-1))*(x - w^2)*(x - w^(-2))*(x - w^4)*(x - w^(-4))
==
p(x) = (x^2 - Ax + 1)*(x^2 - Bx + 1)*(x^2 - Cx + 1)

Obs 1: A, B e C são irracionais;
Obs 2: p(x) é irredutível sobre Q.

[]s,
Claudio.

- Original Message -
From: niski [EMAIL PROTECTED]
To: [EMAIL PROTECTED]
Sent: Monday, May 10, 2004 1:56 PM
Subject: [obm-l] Fatoração II


 Vamos ver agora.
 Fatore x^6 + x^3 + 1

 Obs. Para evitar respostas do tipo
 1*(x^6 + x^3 + 1) ou sobre o que realmente significa fatorar,
 eu cheguei numa expressao do tipo
 (f(x) - Ax + B)(f(x) - Cx + B)(f(x) - Dx + C) onde A,B,C,D sao
 constantes e f é uma funcao...

 Depois eu coloco exatamente qual foi a minha resposta. Mas sera que
 alguem sabe um jeito simples de fazer?

 Abraços

 --
 Niski - http://www.linux.ime.usp.br/~niski

 [upon losing the use of his right eye]
 Now I will have less distraction
 Leonhard Euler

 =
 Instruções para entrar na lista, sair da lista e usar a lista em
 http://www.mat.puc-rio.br/~nicolau/olimp/obm-l.html
 =

=
Instruções para entrar na lista, sair da lista e usar a lista em
http://www.mat.puc-rio.br/~nicolau/olimp/obm-l.html
=


Re: [obm-l] Re: [obm-l] Fatoração II

2004-05-10 Por tôpico niski
É isso mesmo Claudio.
Eu não apelei para a forma exponecial dos complexos. Veja
x^6 + x^3 + 1 = 0
t = x^3
t=-1/2 +- (sqrt(3)/2)i
x = ((|z|)^(1/n))(cos(phi) + isen(phi))
phi = (theta + h2pi)/n
No caso temos
|z| = 1
theta = 2pi/3
n = 3
Assim

h = 0 = phi = 2pi/2
h = 1 = phi = 8pi/9
h = 2 = phi = 14pi/9
Agora considere que para um polinomio do 2 grau

p(x) = [x-(a+bi)][x-(a-bi)]
p(x) = x^2 - 2ax + a^2 + b^2
a = ro*cos(phi) , b = ro*sin(phi)
entao

p(x) = x^2 - 2*ro*cos(phi)*x + ((ro)^2)((sin(phi))^2 + (cos(phi))^2)
p(x) = x^2 - 2*ro*cos(phi)*x + (ro)^2
Acoplando isso para o polinomio em questao
x^6 + x^3 + 1 = (x^2 - 2*cos(2pi/9)x + 1)(x^2 - 2*cos(8pi/9)x + 1)(x^2 - 
2*cos(14pi/9)x + 1)

Cláudio (Prática) wrote:

p(x) = x^6 + x^3 + 1 = (x^9 - 1)/(x^3 - 1)
Ou seja, as raízes de p(x) são as raízes nonas da unidade com exceção de 1,
exp(i*2pi/3) e exp(i*4pi/3).
Seja w = exp(i*2pi/9).
Então as raízes de x^6 + x^3 + 1 são:
w, w^2, w^4, w^(-1), w^(-2) e w^(-4).
w + w^(-1) = 2*cos(2pi/9) = A
w^2 + w^(-2) = 2*cos(4pi/9) = B
w^4 + w^(-4) = 2*cos(8pi/9) = C
Logo:
p(x) = (x - w)*(x - w^(-1))*(x - w^2)*(x - w^(-2))*(x - w^4)*(x - w^(-4))
==
p(x) = (x^2 - Ax + 1)*(x^2 - Bx + 1)*(x^2 - Cx + 1)
Obs 1: A, B e C são irracionais;
Obs 2: p(x) é irredutível sobre Q.


--
Niski - http://www.linux.ime.usp.br/~niski
[upon losing the use of his right eye]
Now I will have less distraction
Leonhard Euler
=
Instruções para entrar na lista, sair da lista e usar a lista em
http://www.mat.puc-rio.br/~nicolau/olimp/obm-l.html
=


[obm-l] Fatoração ( IMO )

2004-05-09 Por tôpico Fabio Contreiras



Alguem tem ideia de como fatorar isso? Um 
Abraço!


(x +y )^7-( x^7 + y^7 
)


[obm-l] Re: [obm-l] Fatoração ( IMO )

2004-05-09 Por tôpico Rafael
Fábio,

Acho pouco provável que esse tipo de exercício tenha caído numa IMO, mas...

(x + y)^7 - (x^7 + y^7) = 7xy(x + y)(x^2 + xy + y^2)^2

Uma identidade semelhante foi usada por Lamé na demonstração do Último
Teorema de Fermat para n = 7.

(x + y + z)^7 - (x^7 + y^7 + z^7) =
= 7(x+y)(x+z)(y+z)[(x^2 + y^2 + z^2 + xy + xz + yz)^2 + xyz(x + y + z)]


Abraços,

Rafael de A. Sampaio





- Original Message -
From: Fabio Contreiras
To: [EMAIL PROTECTED]
Sent: Sunday, May 09, 2004 2:32 PM
Subject: [obm-l] Fatoração ( IMO )


Alguem tem ideia de como fatorar isso? Um Abraço!


( x + y )^7 - ( x^7 + y^7 )



=
Instruções para entrar na lista, sair da lista e usar a lista em
http://www.mat.puc-rio.br/~nicolau/olimp/obm-l.html
=


RE: [obm-l] Fatoração ( IMO )

2004-05-09 Por tôpico Marcelo Souza


From: "Fabio Contreiras" [EMAIL PROTECTED] 
Reply-To: [EMAIL PROTECTED] 
To: [EMAIL PROTECTED] 
Subject: [obm-l] Fatoração ( IMO ) 
Date: Sun, 9 May 2004 14:32:34 -0300 
 
Alguem tem ideia de como fatorar isso? Um Abraço! 
 
 

( x + y )^7 - ( x^7 + y^7 ) 
x^7+y^7=(x+y)(x^6-x^5y+x^4y^2-...-xy^5+y^6)
Daí você põe (x+y) em evidência pra obter:
(x+y)^7-x^7-y^7=(x+y)[(x+y)^6-x^6+x^5y-x^4y^2+...+xy^5-y^6)]
A rigor,já está fatorado, mas, você pode "ajeitar" a expressão dentrodo [].
[]'s, Marcelo.
MSN Messenger: converse com os seus amigos online. Instale grátis. Clique aqui. 
=
Instruções para entrar na lista, sair da lista e usar a lista em
http://www.mat.puc-rio.br/~nicolau/olimp/obm-l.html
=


Re: [obm-l] Fatoração ( IMO )

2004-05-09 Por tôpico Eduardo Henrique Leitner
= (x + y)^7 - (x + y)(x^6 - x^5y + x^4y^2 - x^3y^3 + x^2y^4 - xy^5 + y^6) =
= (x+y)[ (x + y)^6 - (x^6 + y^6 -x^5y - xy^5 + x^4y^2 + x^2y^4 - x^3y^3) ]

aqui tenho uma duvida

o que exatamente significa fatorar? c eh colocar a expressão como sendo o produto de 2 
fatores, essa resposta jah eh valida...

On Sun, May 09, 2004 at 02:32:34PM -0300, Fabio Contreiras wrote:
 Alguem tem ideia de como fatorar isso? Um Abraço!
 
 
 ( x + y )^7 - ( x^7 + y^7 )

=
Instruções para entrar na lista, sair da lista e usar a lista em
http://www.mat.puc-rio.br/~nicolau/olimp/obm-l.html
=


[obm-l] Re: [obm-l] Re: [obm-l] Fatoração ( IMO )

2004-05-09 Por tôpico Fabio Contreiras
Valeu rafael, po então foi lorota do cara que me passou isso :) abraços!
- Original Message - 
From: Rafael [EMAIL PROTECTED]
To: OBM-L [EMAIL PROTECTED]
Sent: Sunday, May 09, 2004 2:55 PM
Subject: [obm-l] Re: [obm-l] Fatoração ( IMO )


 Fábio,

 Acho pouco provável que esse tipo de exercício tenha caído numa IMO,
mas...

 (x + y)^7 - (x^7 + y^7) = 7xy(x + y)(x^2 + xy + y^2)^2

 Uma identidade semelhante foi usada por Lamé na demonstração do Último
 Teorema de Fermat para n = 7.

 (x + y + z)^7 - (x^7 + y^7 + z^7) =
 = 7(x+y)(x+z)(y+z)[(x^2 + y^2 + z^2 + xy + xz + yz)^2 + xyz(x + y + z)]


 Abraços,

 Rafael de A. Sampaio





 - Original Message -
 From: Fabio Contreiras
 To: [EMAIL PROTECTED]
 Sent: Sunday, May 09, 2004 2:32 PM
 Subject: [obm-l] Fatoração ( IMO )


 Alguem tem ideia de como fatorar isso? Um Abraço!


 ( x + y )^7 - ( x^7 + y^7 )



 =
 Instruções para entrar na lista, sair da lista e usar a lista em
 http://www.mat.puc-rio.br/~nicolau/olimp/obm-l.html
 =


=
Instruções para entrar na lista, sair da lista e usar a lista em
http://www.mat.puc-rio.br/~nicolau/olimp/obm-l.html
=


Re: [obm-l] Fatoração ( IMO )

2004-05-09 Por tôpico Johann Peter Gustav Lejeune Dirichlet
O comum na fatoraçao e fazer os termos serem dois
a dois coprimos.
 --- Eduardo Henrique Leitner
[EMAIL PROTECTED] escreveu:  = (x + y)^7 - (x
+ y)(x^6 - x^5y + x^4y^2 -
 x^3y^3 + x^2y^4 - xy^5 + y^6) =
 = (x+y)[ (x + y)^6 - (x^6 + y^6 -x^5y - xy^5 +
 x^4y^2 + x^2y^4 - x^3y^3) ]
 
 aqui tenho uma duvida
 
 o que exatamente significa fatorar? c eh
 colocar a expressão como sendo o produto de 2
 fatores, essa resposta jah eh valida...
 
 On Sun, May 09, 2004 at 02:32:34PM -0300, Fabio
 Contreiras wrote:
  Alguem tem ideia de como fatorar isso? Um
 Abraço!
  
  
  ( x + y )^7 - ( x^7 + y^7 )
 

=
 Instruções para entrar na lista, sair da lista
 e usar a lista em

http://www.mat.puc-rio.br/~nicolau/olimp/obm-l.html

= 

=
TRANSIRE SVVM PECTVS MVNDOQVE POTIRI 

CONGREGATI EX TOTO ORBE MATHEMATICI OB SCRIPTA INSIGNIA TRIBVERE 

Fields Medal(John Charles Fields)
 
N.F.C. (Ne Fronti Crede)



__

Yahoo! Messenger - Fale com seus amigos online. Instale agora! 
http://br.download.yahoo.com/messenger/
=
Instruções para entrar na lista, sair da lista e usar a lista em
http://www.mat.puc-rio.br/~nicolau/olimp/obm-l.html
=


Re: [obm-l] Re: [obm-l] Re: [obm-l] Fatoração ( IMO )

2004-05-09 Por tôpico Fabio Dias Moreira

Fabio Contreiras said:
 Valeu rafael, po então foi lorota do cara que me passou isso :) abraços!
 [...]

Eu acho que você quer o seguinte problema:

(IMO-84) Encontre todos os inteiros a, b tais que ab(a+b) não é múltiplo
de 7 mas (a+b)^7 - (a^7 + b^7) é divisível por 7^7.

[]s,

-- 
Fábio ctg \pi Dias Moreira


=
Instruções para entrar na lista, sair da lista e usar a lista em
http://www.mat.puc-rio.br/~nicolau/olimp/obm-l.html
=


[obm-l] Re: [obm-l] Re: [obm-l] Re: [obm-l] Fatoração ( IMO )

2004-05-09 Por tôpico Rafael
Graças ao Fábio D. Moreira, agora sabemos que a lorota foi parcial...

Como o problema, pelo visto, interessou a várias pessoas da lista, eis a
demonstração que eu havia omitido:

(x + y)^7 - x^7 - y^7 =
= 7x^6y + 21x^5.y^2 + 35x^4.y^3 + 35x^3.y^4 + 21x^2.y^5 + 7x.y^6
= 7xy(x^5 + 3x^4.y + 5x^3.y^2 + 5x^2.y^2 + 3x.y^4 + y^5)
= 7xy[(x+y)(x^4-x^3.y+x^2.y^2-x.y^3+y^4+3xy(x^3+y^3)+5x^2.y^2(x+y)]
= 7xy[(x+y)(x^4-x^3.y+x^2.y^2-x.y^3+y^4+3xy(x+y)(x^2-xy+y^2)+5x^2.y^2(x+y)]
= 7xy(x+y)[x^4-x^3.y+x^2.y^2-x.y^3+y^4+3xy(x^2-xy+y^2)+5x^2.y^2]
= 7xy(x+y)[x^4 - x^3.y - x.y^3 + y^4 + 3xy(x^2 + xy + y^2)]
= 7xy(x+y)[x(x^3 - y^3) - y(x^3 - y^3) + 3xy(x^2 + xy + y^2)]
= 7xy(x+y)[(x-y)(x^3 - y^3) + 3xy(x^2 + xy + y^2)]
= 7xy(x+y)[(x-y)(x-y)(x^2 + xy + y^2) + 3xy(x^2 + xy + y^2)]
= 7xy(x+y)(x^2 + xy + y^2)(x^2 - 2xy + y^2 + 3xy)
= 7xy(x+y)(x^2 + xy + y^2)^2


Abraços,

Rafael de A. Sampaio





- Original Message -
From: Fabio Contreiras [EMAIL PROTECTED]
To: [EMAIL PROTECTED]
Sent: Sunday, May 09, 2004 4:00 PM
Subject: [obm-l] Re: [obm-l] Re: [obm-l] Fatoração ( IMO )


Valeu rafael, po então foi lorota do cara que me passou isso :) abraços!



- Original Message -
From: Rafael [EMAIL PROTECTED]
To: OBM-L [EMAIL PROTECTED]
Sent: Sunday, May 09, 2004 2:55 PM
Subject: [obm-l] Re: [obm-l] Fatoração ( IMO )


 Fábio,

 Acho pouco provável que esse tipo de exercício tenha caído numa IMO,
mas...

 (x + y)^7 - (x^7 + y^7) = 7xy(x + y)(x^2 + xy + y^2)^2

 Uma identidade semelhante foi usada por Lamé na demonstração do Último
 Teorema de Fermat para n = 7.

 (x + y + z)^7 - (x^7 + y^7 + z^7) =
 = 7(x+y)(x+z)(y+z)[(x^2 + y^2 + z^2 + xy + xz + yz)^2 + xyz(x + y + z)]


 Abraços,

 Rafael de A. Sampaio





=
Instruções para entrar na lista, sair da lista e usar a lista em
http://www.mat.puc-rio.br/~nicolau/olimp/obm-l.html
=


Re: [obm-l] Re: [obm-l] Fatoração

2004-03-04 Por tôpico Faelccmm
Ola Rafael,

Voce poderia me dizer como voce fez a divisao de x^3 + y^3 por (x+y) ?
ps: eu ate conheco a divisao pelo metodo da chave, mas nao estou conseguindo neste caso.



Em uma mensagem de 4/3/2004 03:11:46 Hora padrão leste da Am. Sul, [EMAIL PROTECTED] escreveu:



Marcelo,

Uma forma de você conseguir "fatorações mágicas" é raciocinar baseando-se
nas identidades que já conhece, ou ainda, como se se tratassem de equações.
Por exemplo, vou demonstrar a soma de dois cubos:

x^3 + y^3 = 0 == x^3 = -y^3 == x = - y == x + y = 0

Assim, temos que (x+y) é um dos fatores de x^3 + y^3. Se você utilizar o
método da chave (algoritmo de Euclides), dividindo x^3 + y^3 por (x+y),
obterá x^3+y^3 = (x+y)(x^2+y^2-xy).

O seu caso é a soma de três cubos subtraída do triplo do produto das três
variáveis: x^3+y^3+z^3-3xyz. Aqui o raciocínio usado traria:
x^3+y^3+z^3-3xyz = 0 == x^3+y^3+z^3 = 3xyz. Não se obtém uma resposta
direta, mas vale a pena observar que a igualdade deixa uma suspeita: o fator
deve depender de x, y e z. Se você supuser esse fator como (x+y+z) e
utilizar o método da chave novamente, verá que x^3 + y^3 + z^3 - 3xyz =
(x+y+z)(x^2+y^2+z^2-xy-xz-yz).

É bastante claro que isso foi uma tentativa, poderia não ter dado certo.
Fatorar nem sempre é uma tarefa fácil, lembro-me de que Lamé usou a
fatoração de (X+Y+Z)^7-(X^7+Y^7+Z^7) para provar que o Último Teorema de
Fermat é verdadeiro para n = 7. E, acredite, o que ele fez não foi nada,
nada óbvio. Enfim, não há uma receita, mas conhecendo-se (ou
desconfiando-se) de um fator, isso ajuda muito!

Abraços,

Rafael de A. Sampaio




- Original Message -
From: "k4ssmat" [EMAIL PROTECTED]
To: "Lista" [EMAIL PROTECTED]
Sent: Thursday, March 04, 2004 1:56 AM
Subject: [obm-l] Fatoração


Olá,
sou novo na lista, é uma honra conversar sobre matemática com
tantos entendidos do assunto.
Estou estudando para o vestibular e peguei um exercício de
uma das olimpiadas internacionais de matematica (pelo menos é
o que dizia no exercicio), que segue abaixo:

Fatore: x^3 + y^3 + z^3 - 3*x*y*z

Consegui fatorar por diversos caminhos diferentes, mas não
obtendo a resposta. Então peguei a resposta, e a entendi
perfeitamente, o que me faltou foi visão. Pois a resposta foi
apenas um outro caminho que eu ainda não tinha tentado.

Gostaria de dicas, ou exercícios que me ajudem a "enxegar"
essas fatorações. Estou aqui com mais 5 exercícios, sendo 1
deles de uma olimpiada brasileira de matematica, que ainda
não consegui fazer e após sua ajuda, espero conseguir.

Obrigado
Marcelo






[obm-l] Re: [obm-l] Re: [obm-l] Fatoração

2004-03-04 Por tôpico Rafael
Rafael,

Vou tentar desenhar aqui a construção do algoritmo e, por fim, explico o
raciocínio.


   x^3 + y^3  |   x + y
- x^3 - x^2y |¯
|   x^2 - xy + y^2
- x^2*y + y^3  |
x^2*y + x*y^2 |
|
  x*y^2 + y^3   |
- x*y^2 - y^3   |
|
 0 |


O método da chave é um algoritmo que funciona da seguinte forma: o primeiro
termo do dividendo é dividido pelo primeiro termo do divisor, isto é,
x^3/x = x^2. Este resultado (x^2) é multiplicado pelo divisor e subtraído do
dividendo (x^3 + y^3), ou seja, -(x^2*x + x^2*y) = -(x^3 + x^2*y). Após a
subtração, ficamos com -x^2*y + y^3. O processo se reinicia: -x^2*y, que é
agora o primeiro termo do dividendo, é dividido pelo primeiro termo do
divisor: -x^2*y/x = -xy. Este resultado (-xy) é multiplicado pelo divisor e
subtraído do último dividendo (-x^2*y + y^3), ficamos com: x*y^2 + y^3. E
o mesmo se repete: x*y^2 / x = y^2, que é multiplicado pelo divisor,
subtraído do último dividendo (x*y^3 + y^3), resultando em 0 (divisão
exata).

Duas observações são importantes sobre o algoritmo: a cada passo da divisão,
o primeiro termo do dividendo é cancelado, e a divisão continua até que se
obtenha 0 (divisão exata) ou um polinômio de grau menor que o do divisor. E,
por fim, se você prestar atenção ao algoritmo, verá que ele é bem semelhante
ao da divisão euclidiana para os números e a sua justificativa
(demonstração) é basicamente a mesma.

Abraços,

Rafael de A. Sampaio




- Original Message -
From: [EMAIL PROTECTED]
To: [EMAIL PROTECTED]
Sent: Thursday, March 04, 2004 5:40 PM
Subject: Re: [obm-l] Re: [obm-l] Fatoração


Ola Rafael,

Voce poderia me dizer como voce fez a divisao de x^3 + y^3 por (x+y) ?
ps: eu ate conheco a divisao pelo metodo da chave, mas nao estou conseguindo
neste caso.



=
Instruções para entrar na lista, sair da lista e usar a lista em
http://www.mat.puc-rio.br/~nicolau/olimp/obm-l.html
=


[obm-l] Fatoração

2004-03-03 Por tôpico k4ssmat
Olá,
sou novo na lista, é uma honra conversar sobre matemática com 
tantos entendidos do assunto.
Estou estudando para o vestibular e peguei um exercício de 
uma das olimpiadas internacionais de matematica (pelo menos é 
o que dizia no exercicio), que segue abaixo:

Fatore: x^3 + y^3 + z^3 - 3*x*y*z

Consegui fatorar por diversos caminhos diferentes, mas não 
obtendo a resposta. Então peguei a resposta, e a entendi 
perfeitamente, o que me faltou foi visão. Pois a resposta foi 
apenas um outro caminho que eu ainda não tinha tentado.

Gostaria de dicas, ou exercícios que me ajudem a enxegar 
essas fatorações. Estou aqui com mais 5 exercícios, sendo 1 
deles de uma olimpiada brasileira de matematica, que ainda 
não consegui fazer e após sua ajuda, espero conseguir.

Obrigado
Marcelo

 
---
Acabe com aquelas janelinhas que pulam na sua tela.
AntiPop-up UOL - É grátis! 
http://antipopup.uol.com.br


=
Instruções para entrar na lista, sair da lista e usar a lista em
http://www.mat.puc-rio.br/~nicolau/olimp/obm-l.html
=


Re: [obm-l] Fatoração

2004-03-03 Por tôpico Johann Peter Gustav Lejeune Dirichlet
Bem, o negocio e um pouco de pratica.Eu ja
resolvi esse problema junto com a galera da
lista.Mas isso com certeza nao e da IMO.Tente
caçar no arquivo da lista:
http://www.mat.puc-rio.br/~nicolau/olimp/obm-l.html
Depois eu mando umas coisas mais tecnicas sobre.
Bem, mande os outros cinco e a galera ai com
certeza ajuda.


 --- k4ssmat [EMAIL PROTECTED] escreveu: 
Olá,
 sou novo na lista, é uma honra conversar sobre
 matemática com 
 tantos entendidos do assunto.
 Estou estudando para o vestibular e peguei um
 exercício de 
 uma das olimpiadas internacionais de matematica
 (pelo menos é 
 o que dizia no exercicio), que segue abaixo:
 
 Fatore: x^3 + y^3 + z^3 - 3*x*y*z
 
 Consegui fatorar por diversos caminhos
 diferentes, mas não 
 obtendo a resposta. Então peguei a resposta, e
 a entendi 
 perfeitamente, o que me faltou foi visão. Pois
 a resposta foi 
 apenas um outro caminho que eu ainda não tinha
 tentado.
 
 Gostaria de dicas, ou exercícios que me ajudem
 a enxegar 
 essas fatorações. Estou aqui com mais 5
 exercícios, sendo 1 
 deles de uma olimpiada brasileira de
 matematica, que ainda 
 não consegui fazer e após sua ajuda, espero
 conseguir.
 
 Obrigado
 Marcelo
 
  
 ---
 Acabe com aquelas janelinhas que pulam na sua
 tela.
 AntiPop-up UOL - É grátis! 
 http://antipopup.uol.com.br
 
 

=
 Instruções para entrar na lista, sair da lista
 e usar a lista em

http://www.mat.puc-rio.br/~nicolau/olimp/obm-l.html

= 

__

Yahoo! Mail - O melhor e-mail do Brasil! Abra sua conta agora:
http://br.yahoo.com/info/mail.html
=
Instruções para entrar na lista, sair da lista e usar a lista em
http://www.mat.puc-rio.br/~nicolau/olimp/obm-l.html
=


[obm-l] Re: [obm-l] Fatoração

2004-03-03 Por tôpico Rafael
Marcelo,

Uma forma de você conseguir fatorações mágicas é raciocinar baseando-se
nas identidades que já conhece, ou ainda, como se se tratassem de equações.
Por exemplo, vou demonstrar a soma de dois cubos:

x^3 + y^3 = 0 == x^3 = -y^3 == x = - y == x + y = 0

Assim, temos que (x+y) é um dos fatores de x^3 + y^3. Se você utilizar o
método da chave (algoritmo de Euclides), dividindo x^3 + y^3 por (x+y),
obterá x^3+y^3 = (x+y)(x^2+y^2-xy).

O seu caso é a soma de três cubos subtraída do triplo do produto das três
variáveis: x^3+y^3+z^3-3xyz. Aqui o raciocínio usado traria:
x^3+y^3+z^3-3xyz = 0 == x^3+y^3+z^3 = 3xyz. Não se obtém uma resposta
direta, mas vale a pena observar que a igualdade deixa uma suspeita: o fator
deve depender de x, y e z. Se você supuser esse fator como (x+y+z) e
utilizar o método da chave novamente, verá que x^3 + y^3 + z^3 - 3xyz =
(x+y+z)(x^2+y^2+z^2-xy-xz-yz).

É bastante claro que isso foi uma tentativa, poderia não ter dado certo.
Fatorar nem sempre é uma tarefa fácil, lembro-me de que Lamé usou a
fatoração de (X+Y+Z)^7-(X^7+Y^7+Z^7) para provar que o Último Teorema de
Fermat é verdadeiro para n = 7. E, acredite, o que ele fez não foi nada,
nada óbvio. Enfim, não há uma receita, mas conhecendo-se (ou
desconfiando-se) de um fator, isso ajuda muito!

Abraços,

Rafael de A. Sampaio




- Original Message -
From: k4ssmat [EMAIL PROTECTED]
To: Lista [EMAIL PROTECTED]
Sent: Thursday, March 04, 2004 1:56 AM
Subject: [obm-l] Fatoração


Olá,
sou novo na lista, é uma honra conversar sobre matemática com
tantos entendidos do assunto.
Estou estudando para o vestibular e peguei um exercício de
uma das olimpiadas internacionais de matematica (pelo menos é
o que dizia no exercicio), que segue abaixo:

Fatore: x^3 + y^3 + z^3 - 3*x*y*z

Consegui fatorar por diversos caminhos diferentes, mas não
obtendo a resposta. Então peguei a resposta, e a entendi
perfeitamente, o que me faltou foi visão. Pois a resposta foi
apenas um outro caminho que eu ainda não tinha tentado.

Gostaria de dicas, ou exercícios que me ajudem a enxegar
essas fatorações. Estou aqui com mais 5 exercícios, sendo 1
deles de uma olimpiada brasileira de matematica, que ainda
não consegui fazer e após sua ajuda, espero conseguir.

Obrigado
Marcelo

=
Instruções para entrar na lista, sair da lista e usar a lista em
http://www.mat.puc-rio.br/~nicolau/olimp/obm-l.html
=


Re: [obm-l] Fatoração

2004-03-03 Por tôpico Ariel de Silvio



Tb to estudando fatoracao, e to com uns exercicios aqui... to fazendo 
Poliedro em SJCesse especificamente eh o seguintex^3 + y^3 + z^3 - 
3*x*y*zLembre se que (a+b)^3 = a^3 + 3a^2b + 3ab^2 + 
b^3tente "forçar" isso acontecer na expressaox^3 + y^3 + z^3 - 
3*x*y*zx^3 + 3x^2y + 3xy^2 + y^3 + z^3 - 3xyz- 3x^2y- 
3xy^2

as adicoes 
nao alteram em nada, pq vc subtraiu no fim.

(x+y)^3 + z^3 - 3xyz - 3x^2y - 3xy^2

aqui vc precisa lembrar da outra 
identidade

a^3 + b^3 = (a+b)(a^2-ab+b^2)
entao

(x+y)^3 + z^3 - 3xyz - 3x^2y - 3xy^2

(x+y)^3 + z^3 - 3xyz - 3x^2y - 
3xy^2
(x+y+z)[(x+y)^2 - xz - yz+ z^2)] - 3xy(x+y+z)
(x+y+z)(x^2 + 2xy + y^2 - xz - yz - z^2 - 3xy)
(x+y+z)(x^2 + y^2 + z^2 - xy - xz - yz)

prontinho, fatorado...
essa meu professor chamou de "identidade", heheh, tem que estar na ponta do 
lapis sempre...

*** REPLY SEPARATOR ***On 4/3/2004 at 01:56 
k4ssmat wrote:Olá,sou novo na lista, é uma honra conversar 
sobre matemática com tantos entendidos do assunto.Estou 
estudando para o vestibular e peguei um exercício de uma das olimpiadas 
internacionais de matematica (pelo menos é o que dizia no exercicio), 
que segue abaixo:Fatore: x^3 + y^3 + z^3 - 
3*x*y*zConsegui fatorar por diversos caminhos diferentes, mas 
não obtendo a resposta. Então peguei a resposta, e a entendi 
perfeitamente, o que me faltou foi visão. Pois a resposta foi 
apenas um outro caminho que eu ainda não tinha 
tentado.Gostaria de dicas, ou exercícios que me ajudem a 
"enxegar" essas fatorações. Estou aqui com mais 5 exercícios, sendo 1 
deles de uma olimpiada brasileira de matematica, que ainda não 
consegui fazer e após sua ajuda, espero 
conseguir.ObrigadoMarcelo 
---Acabe com aquelas janelinhas que pulam na sua 
tela.AntiPop-up UOL - É grátis! 
http://antipopup.uol.com.br=Instruções 
para entrar na lista, sair da lista e usar a lista 
emhttp://www.mat.puc-rio.br/~nicolau/olimp/obm-l.html=



[obm-l] Fatoração de Polinômios!!!

2003-11-06 Por tôpico Carlos Alberto


Queria saber se existe algum metódo simples para fatorar.
Polinômios de grau "n" sendo que n=2!!!
Por exemplo como eu posso fatorar "x^2 - 4x + 1"
Quais os metódos para fatoração de polinômios, existem vários. 
Vocês podem estar me passando?Yahoo! Mail - 6MB, anti-spam e antivírus gratuito. Crie sua conta agora!

Re: [obm-l] Fatoração de Polinômios!!!

2003-11-06 Por tôpico Eduardo Henrique Leitner
o unico mehtodo que eu conheço eh iguala-lo a:

x^2 - 4x + 1 = (x - a)(x - b)

em que a e b sao as raizes da equação, daih desenvolve-se:

= x^2 -(a+b)x + ab

com isso vc deduz as relações de girard e obtem um sistema:

a + b = 4
ab = 1

daih eh soh acha a solução do sistema, você sempre achará um sistema de n equações com 
n incógnitas, mas esse método nao adianta em muita coisa porque para resolver esse 
sistema você vai ter que resolver a equação x^2 - 4x + 1 = 0 de qualquer maneira...:

a+b = 4 = a = 4 - b
ab = 1 = (4-b)b = 1 = 4b - b^2 = 1 = b^2 - 4b + 1 = 0


te garanto que se descobrirem um método simples e eficiente de fatoração veremos uma 
revolução no campo da matemática... 

[]'s

On Thu, Nov 06, 2003 at 06:52:09AM -0300, Carlos Alberto wrote:
 
 Queria saber se existe algum metódo simples para fatorar.
 
 Polinômios de grau n sendo que n=2!!!
 
 Por exemplo como eu posso fatorar x^2 - 4x + 1
 
 Quais os metódos para fatoração de polinômios, existem vários. 
 
 Vocês podem estar me passando?
 
 
 
 
 -
 Yahoo! Mail - 6MB, anti-spam e antivírus gratuito. Crie sua conta agora!
=
Instruções para entrar na lista, sair da lista e usar a lista em
http://www.mat.puc-rio.br/~nicolau/olimp/obm-l.html
=


Re: [obm-l] Fatoração

2002-07-03 Por tôpico Arnaldo






Olá amigos , será que alguém poderia me ajudar nessa dae ?

O valor de n que satisfaz á igualdade (anexei a equação) é:


Só dar uma idéia , porque não consigo visualizar nenhuma saída.
Um abraço.
Rick


  
 |-=Rick-C.R.B.=- |
 |ICQ 124805654   |
 |e-mail [EMAIL PROTECTED]  |
  


--
Use o melhor sistema de busca da Internet
Radar UOL - http://www.radaruol.com.br

faca x=(17*5^1/2 + 38 )^1/n e y=(17*5^1/2 - 38)^1/n. Note que 
x+y = 20^1/2 e x*y=1, logo vc tem uma equaçao do segundo grau que resolvendo,
encontramos x= 5^1/2 +2 e entao é fácil ver que n dever ser igual a 3.



Attachment: 
equação.gif=

Instruções para entrar na lista, sair da lista e usar a lista em
http://www.mat.puc-rio.br/~nicolau/olimp/obm-l.html
O administrador desta lista é [EMAIL PROTECTED]
=



http://www.ieg.com.br
=
Instruções para entrar na lista, sair da lista e usar a lista em
http://www.mat.puc-rio.br/~nicolau/olimp/obm-l.html
O administrador desta lista é [EMAIL PROTECTED]
=



  1   2   >